AP BIOLOGY FRQ

Pataasin ang iyong marka sa homework at exams ngayon gamit ang Quizwiz!

Morphological data vs Amino acid sequence

Amino acid/ molecular data is more accurate Because morphology may be similar even if they have differences in amino acid/DNA sequences Molecular data directly show genetic make up/ reveal evolution

Geologic events such as mountain building can directly affect biodiversity. The Sino‑Himalayan fern, Lepisorus clathratus is widely distributed on the Qinghai‑Tibetan plateau. If geologic events lead to a series of new mountain ranges that divide the plateau into several discrete valleys, which of the following best describes how the L. clathratus population would be affected?

Geographic isolation will create separate gene pools, leading to speciation over time.

What are the substrates and products of photosynthesis?

Photosynthesis uses sunlight, carbon dioxide, and water as substrates. it produces oxygen and glucose.

Explain how the two above processes (photosynthesis and cellular respiration) are connected to each other.

Photosythesis is the opposite reation of cellular respiration. Both make energy for the cell. Photosythesis produces glucose and cellular respiration breaks it down

Identify and describe the properties of life. Why and how does passive transport occur across membranes?

Plasma membranes are selectively permeable; they allow some substances to pass through, but not others. ... The most direct forms of membrane transport are passive. Passive transport is a naturally-occurring phenomenon and does not require the cell to exert any of its energy to accomplish the movement.

Because cheetahs, Acinonyx jubatus, experienced a bottleneck event about ten thousand years ago, living cheetahs exhibit very little genetic variation compared to other living species of large cats. Which of the following principles best justifies a claim that cheetahs, compared to other large cats, have a relatively low resilience to environmental perturbations?

Populations with little genetic diversity are less likely to contain individuals that can withstand different selective pressures.

Which of the following best describes the processes occurring between the mouse populations and their environments?

Dark color is selected for in mice from the West, Mid, and East populations.

How are complex macromolecule polymers synthesized from monomers?

Dehydration Synthesis.

What is the difference between kinetic and potential energy?

Potential Energy is the stored energy in an object or system because of its position or configuration. Kinetic energy of an object is relative to other moving and stationary objects in its immediate environment.

Some researchers claim that the decrease in plant species richness of this woodlot is unrelated to human activities over the last forty years. Which of the following would be an appropriate alternative hypothesis?

During the last few decades, tall buildings have been built around the woodlot, and species 3, 4, and 5 are shade‑intolerant plants

Compare the roles of general and specific transcription factors in regulating gene expression.

General transcription factors deal with the transcription of promoters of all genes Specific transcription factors bind to certain elements on specific genes and can increase or decrease transcription using activators or repressors.

Explain how the amino acid substitution shown in Figure 1 is most likely to affect the function of Receptor X.

The response indicates that because function is determined by shape, the function of receptor X is likely to change OR that the structural change might alter the binding of the molecule to receptor X OR that the structural change might alter the ability of receptor X to cause a cellular response.

Describe the role of carbon in biological systems.

The response indicates that carbon is used to build biological macromolecules such as carbohydrates, proteins, nucleic acids, and lipids.

Based on the model, explain whether the molecules shown crossing the membrane require energy to do so.

The response indicates that energy is not required for the substances to cross the membrane because the substances are being passively transported from a region of higher concentration to a region of lower concentration.

(a) Describe one measure of evolutionary fitness.

The response indicates that evolutionary fitness is usually measured by reproductive success or that it is measured by the number of offspring produced.

As a cell begins the process of dividing, its chromosomes become shorter, thicker, and individually visible in an LM. Explain what is happening at the molecular level.

each chromosome consists of one long DNA molecule attatched to numerous protein molecules a combination called chromatin, as a cell begins division each chromosome becomes condensed as its diffuse mass of chromatin coils up

endocytosis/exocytosis

endo: cell brings in materials exo: cell transports vesicles to release its contents outside, recycles materials out of cell thru membrane phagocytosis: cellular eating, endo, large molecule to food vacuole

structure of enzyme as inferred from experiment

enzymatic activity constant- enzyme structure has not changed enzymatic activity low- enzyme structure has changed, probably denatured

factors that affect enzymes

enzyme concentration, temperature, pH, substrate concentration, salinity, activators, inhibitors

active site

enzyme's catalytic site; substrate fits into active site

chemical structure of an enzyme to its specificity and catalytic activity

enzymes are typically proteins w a specific active site that accepts substrates of a specific shape, activation energy- enzyme needs to surpass to start chemical reaction; induced fit- chemical groups of active site brought into positions that enhance their ability to catalyze reaction and make it go faster

conditions for Hardy-Weinberg equilibrium

equal reproductive success

fossil evidence for evolution

flat head

evolutionary significance of organizing genes into chromosomes

genetic diversity: independent assortment (homologous chromosomes positioned independently so they separate into daughter cells separately), crossing over (genetic rearrangement btwn non-sister chromatids involving the exchange of DNA segments) lead to variation in gametes complexity: many genes on a chromosome, new genes stability: :)

relationship btwn metabolism and oxygen consumption, temperature of metabolism for each variety of seeds

greater consumption of o2, the greater the metabolism; temperature increases --> metabolism increases too but variety B has a higher rate at both temps; overall the temp increases enzymatic reaction with higher temps

Moss (Alterations of Generations)

haploid gametophyte is dominant stage, zygote is retained in female gametophyte

Hardy-Weinberg p2

homozygous dominant

Hardy-weinberg q2

homozygous recessive

(c) Use the template of a replication fork to draw arrows that represent both continuous and discontinuous DNA synthesis. Draw one long arrow to show continuous DNA synthesis and three arrows to show discontinuous DNA synthesis. The arrows should point in the direction of nucleotide addition, and the three arrows showing discontinuous synthesis should be numbered 1, 2, and 3, in the order of fragment synthesis.

https://us-static.z-dn.net/files/d8c/74c8142b24792c373591f814b5976256.jpg

choose a sexual organism reproduction, advantages

humans- egg from mother sperm from father combine in fertilization to produce a zygote, or fertilized egg, that has genes from the mother and father advantages: offspring genetically unique from parents, increasing genetic diversity and variation

choose an asexual organism reproduction, advantages

hydra- budding (bud, or a mass of mitotically dividing cells, develops into a small hydra that will detach form the parent) advantage: genetic info is identical

feedback inhibition

regulation and coordination of production -product is used by the next step in the pathway -final product is an allosteric inhibitor of the earlier step -results in no unnecessary accumulation of product -synthesis of isoleucine from threonine: isoleucine becomes allosteric inhibitor of the first step in the pathway and as the product accumulates it collides with the enzyme more often than the substrate does

structure and function of eukaryotic chromosomes

sister chromatids: identical dna, come in pairs, distribute one copy of each to a new cell centromeres: hold sister chromatids together, a region of specific DNA sequences kinetochores: disc-shaped proteins that allow for spindle attachment

Rough ER vs Smooth ER function

smooth er lacks ribosomes rough er has ribosomes smooth er synthesizes lipids produces steroids detoxifies drugs and poisons like in liver cells stores calcium ions the rough er makes secretory protiens in the ribosomes, usually glycoproteins membrane factory for the cell

non-competitive inhibitor

binds at a site other than the active site -allosteric inhibitor binds to allosteric site -causes the enzyme to change shape so the active site is no longer a functional binding site (makes enzyme inactive) -anticancer drugs stop DNA production and division of more cells -cyanide poisoning inhibits enzyme in cellular respiration - -stops production of ATP

biotic and abiotic factors in ecological succession

biotic: competition (lower reproductive rates), predation (predators expand their selection of prey as population increases- preference for a specific prey) abiotic: temperature (affects biological like cell rupturing or denaturing of proteins), sunlight (provides energy for photosynthesis, limited light can limit the distribution of photosynthetic organisms)

competitive inhibitor

competes with the substrate for the the active site -penicilin: blocks enzyme bacteria use to build cell walls -disulfiram antabuse: treats chronic alcoholism by blocking enzyme that breaks down alcohol and gives you a severe hangover/vomiting after drinking -can be overcome by increasing substrate concentration bc the substrates will outcompete the inhibitor for the substrate

three factors that might cause the fluctuations as seen in phase B

competition: limit the resources and cause reproductive rates to drop reproduction: after a negative event, reproduction increases disease: decreases population size and cause many individuals to die OVERALL, THE POPULATION WILL RISE AND FALL IN A CYCLE

induced fit model

conformational change in the shape of an enzyme's active site that enhances the fit between the active site and its substrate(s)

allosteric regulation

conformational changes by regulatory molecules -inhibitors: keeps enzyme in inactive form -activators: keep enzyme in active form

ways bacteria become resistant to antibiotics

conjugation // when one bacteria gives its DNA to another bacteria- if the first bacteria is resistant it can give the second bacteria resistance

controlled experiment for diversity of plant species in newly abandoned field

hypothesis: large herbivores will decrease diversity independent variable: herbivores dependent variable: # of diff species of plants set up two fields one w herbivores and one w/o so u can see the difference in plant diversity and how it would've been; keep sunlight, temp, soil composition, and precipitation the same (how u would really do this in reality idk)

2007 Membranes are essential components of all cells B. Explain how membranes participate in THREE of the following biological processes: iii. Chemiosmotic production of ATP iv. Intercellular signaling

iii. Electron transport chain in membrane pumps H+ across membrane, Membrane infolding increases surface area iv. release of chemical signals by exocytosis, ligand-gated ion channels opening/closing

irreversible inhibition

inhibitor permanently binds to enzyme -competitor: permanently binds to active site -allosteric: permanently binds to allosteric site and changes shape of protein

activation energy

initial input of energy required to break down large molecules -amount of energy needed to destabilize the bonds of a molecule

macromolecules in plasma membrane (structure and function)

integral proteins: penetrate hydrophobic interior of lipid bilayer and span whole membrane and form a channel, provide hydrophilic channel (molecules pass thru membrane or have an active site where they accept enzymes and other substrates); made of polypeptides of aa cholesterol: steroid with ring structure, amphipathic molecule embedded in bilayer; moderates fluidity and stabilizes membrane phospholipids: phosphate + glycerol + 2 fatty acids; forms lipid bilayer for the membrane (amphipathic- hydrophilic head faces the exterior and hydrophobic is interior); selectively permeable, membrane fluidity

active transport

ion or molecule passes thru a membrane against its concentration gradient (low to high), REQUIRES ENERGY, occurs in transmembrane proteins, plants: proton pump; animals: Na+/K+ pump that establishes polarity

how can natural selection lead to antibiotic resistance

it can give bacteria selective advantage- it allows bacteria to have higher survival rate

how can natural selection lead to antibiotic resistance

it can increase the reproductive fitness of bacteria- ability to leave most offspring- bacteria resistant to antibiotics living among antibiotics are more fit than bacteria that aren't

Complete the table to represent the relationship between volume and surface area of a spherical structure surrounded by a membrane

just remember that the ratio is 1:3 and 1:1

conditions for Hardy-Weinberg equilibrium

large population

Similarities between transcription and translation

mRNA is involved in both processes and they both aid in the synthesis of proteins

enzyme concentration

more enzymes = more frequently collide with substrate -reaction rate levels off as substrate becomes limiting factor

substrate concentration

more substrate = more frequently collide with enzyme -reaction rate levels off as all enzyme is saturated (all active sites are engaged)

What's good about preventing self-fertilization

more variation

ways bacteria become resistant to antibiotics

mutation // bacteria is mutated so that it is resistant to antibiotics

conditions for Hardy-Weinberg equilibrium

no mutations

genetic basis of abnormal hemoglobin

point mutation in DNA, where a base substitution leads to a diff aa in hemoglobin (glutamate to valine)

Flowers; how to bring male and female gametes together

pollination

exponential (r) and logistic (k) reproductive strategies

r-strategists: type III survivorship curve- many offspring, but few offspring survive into adulthood; little investment by parents k-strategists: type I survivorship curve- few offspring but majority survive into adulthood; high investment by parents NOW DRAW THE GRAPHS YOU KNOW THEM

conditions for Hardy-Weinberg equilibrium

random mating

substrate

reactant which binds to enzyme -enzyme-substrate complex: temporary association

endomembrane system

regulates proteins traffic and performs metabolic functions nucleaur envelope, endoplasmic reticulum, golgi apparatus, lysosomes, vesicles and vacuoles, plasma membrane related through physical continuity or vesicles

Identify and define the levels of structure found in a protein

Primary structure is a linear chain of amino acids. Secondary structure is a combination of polar and nonpolar (hydrophobic/hydrophillic) acids pair up Tertiary structure is where hydrophobic acids are in the center of the structure and hydrophilic on the outside. Quartenary structure is multiple proteins in one structure

Explain the fluid mosaic model

States that the phospholipid bilayer behaves more like a liquid than a solid. Fluid bc the phospholipids and proteins move side to side like a liquid Mosaic bc when viewed from above, the scattered protein molecules look like a mosaic.

differences between transcription and translation

transcription is in nucleus while translation is in the cytoplasm, transcription creates mrna while translation creates polypeptides, template for transcription is dna while for translation it is mrna.

Functions of Golgi Apparatus

warehouse for recieving sorting and shipping and manufacturing. products of ER are modified and stored then sent to other destinations. consists of cisternae the cis face recieves vesicles with ER products and the trans side dispatches the vesiclesw

As solute potential increases...

water potential decreases

As pressure increases...

water potential increases

enzymes

work with a specific substrate (chemical fit between active site and substrate)

which type of microscope would you use to study (a) the changes in shape of a living white blood cell and (b) the details of surface texture of a hair

(A) light microscope (B) scanning electron microscope

(c) (i) Based on the data, identify the population that is likely to have the highest frequency of heterozygotes. (c) (ii) Assuming random mating, calculate the frequency of animals in population 1 that carry both alleles 1 and 3.

(c) (i) The response indicates that population 3 is likely to have the highest frequency of heterozygotes (because in population 3 all alleles are present at closest to the same frequencies). (c) (ii) The response indicates that the frequency of animals in population 1 that carry both alleles 1 and 3 is 0.3 or 30%. The calculation is: 2(0.60×0.25)=2×0.15=0.30

(d) (i) A sudden event drastically changes the habitat of each population such that most of the normal vegetation and insects are replaced by other vegetation and insects. One of the scientists claims that population 2 will be the most severely affected. Predict the most likely effect of the event on population 2. (d) (ii) Provide reasoning to justify your prediction.

(d) (i) The response indicates that the number of animals in population 2 will decline. Many of the animals in population 2 are likely to have trouble finding and identifying sources of food. (d) (ii) The response indicates that population 2 has the least genetic diversity of alleles of the opsin gene. The primates in this population will primarily see one wavelength of light/color, that dictated by allele 1, and so may have difficulty distinguishing other new and different forms of food.

1. 1994 Genetic variation is the raw material for evolution. a. Explain three cellular and/or molecular mechanisms that introduce variation into the gene pool of a plant or animal population. b. Explain the evolutionary mechanisms that can change the composition of the gene pool.

) The overall resistance to drugs in bacteria increases. It is possible to observe the plasmids exchange between bacteria. We know that plasmids are able to ) The overall resistance to drugs in bacteria increases. It is possible to observe the plasmids exchange between bacteria. We know that plasmids are able to determine resistance to antibiotics. Thus, we can prove that bacteria change in a way that benefits them over time - or, in other words, evolve. b) Biochemistry of different organisms are similar, there are no radical differences, so we can safely claim that certain creatures are descendants of others. c) In fossils we can find organisms that posses traits of two different groups of organisms (e.g. Archeopteryx, Seymouria, Ichtyostega). This is a proof that there indeed are groups that are "links" between different species. resistance to antibiotics. Thus, we can prove that bacteria change in a way that benefits them over time - or, in other words, evolve. b) Biochemistry of different organisms are similar, there are no radical differences, so we can safely claim that certain creatures are descendants of others. c) In fossils we can find organisms that posses traits of two different groups of organisms (e.g. Archeopteryx, Seymouria, Ichtyostega). This is a proof that there indeed are groups that are "links" between different species.

List at least three different properties that have been acquired by crop plants via genetic engineering.

- pesticide resistance - herbicide resistance - delayed ripening - drought resistance

ENZYME -describe the smaller units -their assembly into the larger structures -one major function of these larger, organized structures.

-Made up of amino acids -Amino acids link together via peptide bonds resulting in a polypeptide or protein -Enzymes lower the activation energy

A scientist wishes to provide experimental evidence to support the model shown in Figure 1 by demonstrating the ability to synthesize an RNA molecule. Which of the following is an alternative hypothesis that can be tested to support the RNA World Hypothesis?

A mixture of ribonucleotides such as adenine and uracil will spontaneously form single-stranded chains of RNA .

What is the structure and role of DNA?

A nucleic acid that contains the genetic instructions for the development and function of living things. All known cellular life and some viruses contain DNA. The main role of DNA in the cell is the long-term storage of information.

What is the basic structure of a steroid, and what are examples of their functions?

A steroid is a biologically active organic compound with four rings arranged in a specific molecular configuration. Steroids have two principal biological functions: as important components of cell membranes which alter membrane fluidity; and as signaling molecules.

The chemical composition and levels of structure of proteins

A string of amino acids held together by a peptide bond is a protein. Every protein has free amino and a free carboxyl end. The primary structure is the order of the amino acids in the protein. The secondary structure has the long string of amino acids coiling into an alpha helix or forming a beta-pleated sheet. The tertiary structure is composed of the beta-pleated sheets and the alpha helices folding together. The quaternary structure is the structure that results of the assembly of several polypeptide to make an unique functional protein.

2002-B Chart A. Describe and explain the observed results

A. - population reaches an equilibrium, -virus attacks bacteria, bacteria decline while virus multiplies -resistant bacteria survive to reproduce; virus number drops, -virus multiply only in non-resistant cells or a lysogenic situation develops

How is energy released through the hydrolysis of ATP?

ATP hydrolysis is the catabolic reaction process by which chemical energy that has been stored in the high-energy phosphoanhydride bonds in adenosine triphosphate (ATP) is released by splitting these bonds, for example in muscles, by producing work in the form of mechanical energy.

Why is ATP considered the energy currency of the cell?

ATP is called the energy currency of the cell because it is the molecule that all living organisms use for energy.

What are the differences between anabolic and catabolic pathways?

Anabolic pathways are those that require energy to synthesize larger molecules. Catabolic pathways are those that generate energy by breaking down larger molecules. Both types of pathways are required for maintaining the cell's energy balance. ... Catabolic pathways break large molecules into small pieces.

2002-B B. Discuss the infection cycle of a DNA virus from attachment to lysis.

B. -attachment to host cell membrane -injection of DNA -synthesis of proteins -assembly of viruses -new viruses released from the host cell

2011-B B. Explain the role of THREE of the following in mitosis or cytokinesis i. Kinetochores ii. microtubules iii. motor proteins iv. actin filaments

B. i. kinetochores: located in centromeres of condensed chromosomes. ii. microtubules: fundamental structural element of spindle apparatus iii. motor proteins: in cleavage furrow, generate force to pinch cell in two

DNA sequences can act as "tape measures of evolution." Scientists analyzing the human genome sequence were surprised to find that some of the regions of the human genome that are most highly conserved (similar to comparable regions in other species) don't code for proteins. Propose a possible explanation for this observation.

Because of the segments' conservative nature, it is likely that the genes are very important and not something you would want mutated. They would most likely control the development of appendages on an organism. So a mutation in those regions could have a severe negative impact on an organism's ability to function and survive.

Bromelain works by breaking the enzymes that cause browning into smaller molecules. Explain how the reaction that breaks up the enzymes occurs.

Bromelain separates the amino acids in the enzymes that cause browning by breaking the peptide bonds.

2011-B C. Describe how the cell cycle is regulated and discuss ONE consequence of abnormal regulation

C. -Regulation: hormones, growth factor control of cell cycle activity, contact inhibition of mitosis -Consequence: uncontrolled cell division = cancer

2002-B C. Describe how the genome of a retrovirus like HIV becomes incorporated into the genome of the cost cell

C. -retrovirus: RNA virus -use of reverse transcriptase to create DNA from RNA -single strand to double strand conversion -glycoprotein envelope of virus attaches to membrane of the host cell, membranes fuse together, viral components release and capsid degraded, reverse transcriptase, synthesis of viral DNA (provirus), vesicles transport glycoproteins to cell plasma membrane, capsids assembled, new viruses bud off.

13. Describe the light reactions of photosynthesis and, for both a C3 and a C4 plant, trace the path of a carbon dioxide molecule from the point at which it enters a plant to its incorporation into a glucose molecule. Include leaf anatomy and biochemical pathways in your discussion of each type of plant.

C3 Plants: -PS I and PS II C4 Plants: -Bundle Sheath Cells

9. Discuss how C4 and CAM photosynthesis improve upon C3 photosynthesis. Include both advantages and disadvantages of the processes. Describe appropriate leaf anatomy to complete your discussion.

C4: bundle sheath cells CAM: malic acid Advatages: -Doesn't become shriveled up and lose H2O Disadvantages: -Not as efficient, might lose H2O during the day

What limits transpiration rates

CAM photosynthesis, and storing water in a modified stem

What limits the size of a cell?

Cell size is limited by a cell's surface area to volume ratio.

determine the difference between cofactors and coenzymes

Cofactors are inorganic (do not contain carbon). Coenzymes are organic (contain carbon).

What is the structure and roles of RNA?

Complex compound of high molecular weight that functions in cellular protein synthesis and replaces DNA (deoxyribonucleic acid) as a carrier of genetic codes in some viruses.

Of the following, which is the best next step for the researchers to best determine the long-term effectiveness of these insecticides with regards to the development of resistance?

Continue treating the three insect populations with the same insecticides and determine whether their efficiencies decline significantly after several generations.

Based on the phylogenetic relationships shown in Figure 1, which of the following best explains the similarities exhibited by whales, bony fishes, and cartilaginous fishes?

Convergent evolution, because these organisms do not share a recent common ancestor.

14. Genetic variation is the raw material for evolution. Explain three cellular and/or molecular mechanisms that introduce variation into the gene pool of a plant or animal population.

Crossing over is in prophase Uno and occurs at chiasmata. Chromosomes make out and trade pieces of themselves. Independent assortment is that chromosomes will line up along the metaphase plate randomly and split independent dently of other chromosomes. Random fertilization is when a random sperm from your dad fertilized a random efgg from your mom.

Flowers (Alterations of Generations)

Diploid sporophyte is dominant stage, zygote is in a seed

Fertilization in Flowering plants

Double fertilization

What is the difference between light microscopy and electron microscopy?

Electron microscopes differ from light microscopes in that they produce an image of a specimen by using a beam of electrons rather than a beam of light. Electrons have much a shorter wavelength than visible light, and this allows electron microscopes to produce higher-resolution images than standard light microscopes.

What is "energy"?

Energy is the quantitative property that must be transferred to an object in order to perform work on, or to heat, the object.

How do enzymes function as molecular catalysts?

Enzymes lower the activation energy of the reaction but do not change the free energy of the reaction. A substance that helps a chemical reaction to occur is called a catalyst, and the molecules that catalyze biochemical reactions are called enzymes.

What is the difference between endergonic and exergonic reactions?

In an exergonic reaction, energy is released to the surroundings. The bonds being formed are stronger than the bonds being broken. In an endergonic reaction, energy is absorbed from the surroundings.

What are the characteristics shared by the natural sciences?

Natural science are different fields of science that relates to the physical world and its phenomena and processes.

Students created a model showing one possible mechanism for speciation in which a geographic barrier separated one large population into two smaller populations, A and B. Over time, populations A and B developed into two distinct species. The model is shown in Figure 1.

Natural selection acted on variation that was present in both populations A and B, and the lack of gene flow eventually led to the formation of two new species.

How flowers reproduce

Pollination and Double Fertilization

What are the four levels of protein organization?

Primary, secondary, tertiary, and quaternary.

Embryo Protection in Flowers

Seed coat, and endosperm

What is the role of enzymes in metabolic pathways?

Some enzymes help to break down large nutrient molecules, such as proteins, fats, and carbohydrates, into smaller molecules. This process occurs during the digestion of foodstuffs in the stomach and intestines of animals.

2010 C. Compare the cell-signaling mechanisms of steroid hormones and protein hormones

Steroid hormones: mechanism is to alter gene expression in the target cell, actions are slow by sustained Protein hormones: hormone is the ligand and the first messenger, actions are brief but dramatic

Based on the data in Table 1, which of the following best describes the relationship between the MC1R gene and coat color in the Carrizozo, New Mexico, rock pocket mouse population?

The MC1RD allele is not responsible for dark coat color in the Carrizozo population, so the dark coat color is likely a result of convergent evolution.

What are the products of the citric acid cycle?

The citric acid cycle is a series of reactions that produces two carbon dioxide molecules, one GTP/ATP, and reduced forms of NADH and FADH2.

How do the molecular components of the membrane provide fluidity?

The integral proteins and lipids exist in the membrane as separate but loosely-attached molecules.

What is transcription?

The synthesis of RNA on a DNA template

2 Mechanisms of pollen transfer

Wind for lightweight pollen, and animals for colors and smells

what is an energy currency

a way to pass energy around

2. The molecule ATP plays a central role in metabolism. Describe the structure of the ATP molecule and explain how its structure is responsible for its high free energy.

a) ATP is adenosine triphosphate is made of the nitrogenous base adenine, a rbose sugar, and three phosphate groups. b) The bond is held within phosphate groups and when the bond is broken free energy is released

metabolism

all of the chemical reactions that occur within an organism, interactions bt molecules

Identify the type of monomer of which this enzyme, bromelain is composed.

amino acids

metabolic pathway

begins with a specific molecule, which is then altered in a series of defined steps, resulting in a certain product

fossil evidence for evolution

bone structure // one bone, two bones, lotsa bones

How Ferns reproduce

flagellated sperm swim to egg and fuse to make zygote

Hardy-Weinberg 2pq

heterozygous

conditions for Hardy-Weinberg equilibrium

no migration/gene flow

catalyst

proteins that reduce the activation energy needed for a reaction and increase the rate of it without being consumed -highly specific

generalized hormone-signaling pathway

step 1: reception- ligand binds to a receptor protein, and a chemical signal is detected step 2: transduction- series of steps that convert the signal to a form that can bring out a SPECIFIC CELLULAR RESPONSE, each relay molecule transduces the signal from the plasma membrane to the next molecule until it reaches the nucleus step 3: response- transduced signal produces a specific cellular response, expression of target cells will either be stimulated or repressed, according to the signal

Males with Kartagener's syndrome have sperm that fail to swim. Which of these proteins would most likely be involved in the defect?

such indivi have defects in movement of cilia and flagella, so the sperm cant move because of malfunctioning or nonexistent flagella, and the airways are compromised becaise cilia in the trachea malfunction so mucus cannot be cleared from the lungs

fossil evidence for evolution

swiveling neck

image a protein that function in the ER but requires modification in the golgi apparatus before it can achieve that function

the MRNA is synthesized in the nucleus and then passes out through a nuclear pore to be translated on a bound ribosomes attatched to the rough er the protein is synthesized into the lumen of the ER and modified a transport vesicle carries the protein to the golgi apparatus after further modification, another transport vesicle carries it back to the ER where it will perform its cellular function,

ways bacteria become resistant

transduction // DNA passed from bacteria to bacteria through virus, so one virus can pick up a gene of antibiotic resistance from a bacteria, and when the virus goes to infect another bacteria it can give the second bacteria the antibiotic resistance gene

ways bacteria become resistant to antiobiotics

transformation // bacteria takes in genes from their surroundings that allow them to become resistant to antibiotics

Describe Pollination

tube cell forms, 2 sperm travel down tube

Mechanisms that disperse offspring (MOSS)

water

Moss; how to bring male and female gametes together

water for flagellated sperm to swim in

(b) (i) Using the template, construct an appropriately labeled graph to represent the data in Table 1. (b) (ii) Based on the data, determine whether there is a significant difference in the frequency of EACH allele among the three primate populations.

(b) (i) The sketched bars meet all of the criteria below. Correct axis labeling Correct scale and unit Correctly plotted bar graph The y-axis should be labeled "Frequency." In the best form of the graph, the x-axis is labeled "Alleles," and the frequency bars of a particular allele in each population are adjacent. (b) (ii) The response indicates that based on the lack of overlap among error bars for each allele, there IS a significant difference in the frequency of EACH allele among the three populations.

Alterations of Generations

Sporophytes make spores, spores develop into gametophytes, gametophytes make gametes, and gametes fuse to make zygote.

What are some potential difficulties in using plasmid vectors and bacterial host cells to produce large quantities of proteins from cloned eukaryotic genes?

- Bacteria have less DNA than euk cells - Euk genes are too large to be incorporated into euk plasmids - Bacteria cells cannot process RNA transcription (DNA to RNA) - Bacteria lack enzymes for translation which is RNA into protein

In a further experiment, the researchers add a compound to the cell growth medium that both binds and releases protons (H+) and also passes through lipid membranes. Predict the effect of this added compound on ATP production by the cells. Justify your prediction.

-The response indicates that the amount of ATP produced will decrease OR that no ATP will be produced. -The response indicates that the compound is more likely to bind protons where the protons are in greater concentration, in the intermembrane space. It also does not indicate that if the compound then carries the protons across the inner membrane to the matrix, there will no longer be a proton concentration/pH/electrochemical gradient across the inner membrane, and protons will not flow through and power the ATP synthase enzymes.

In the Arizona populations, the melanocortin-1 receptor (MC1R) gene is responsible for coat color phenotype (light or dark). The MC1RD allele, which results in a dark coat, is dominant to the d allele. Researchers collected samples of mice from all six study sites and sequenced the MC1R gene. They also collected mice with dark coats from a location in New Mexico, the Carrizozo lava flow, to determine if the MC1Rgene was also responsible for the dark coat colors observed in that population. The rock pocket mouse population in Carrizozo is isolated from the populations in Arizona. Table 1 presents the genotypic data from sampled mice from all seven locations. Which of the following is closest to the frequency of the MC1RD allele in the Tule Mountain population?

0.03

In addition to the ABO blood groups, humans have an MN blood type system in which the M and N alleles are co-dominant to one another. A group of scientists found that in one population the frequency of the M allele was 0.82, and the frequency of the N allele was 0.18. Assuming that the population is in Hardy‑Weinberg equilibrium, which of the following is the expected frequency of individuals with the genotype MN?

0.295

16. Describe in detail the process of meiosis as it occurs in an organism with a diploid chromosome number of 4 (2n = 4). Include labeled diagrams in your discussion. Indicate when and how each of the following occurs in meiosis. 1. Crossing over 2. Nondisjunction

1 point: occurs during prophase I of meiosis I 1 point: homologous chromosomes synapse and can exchange genetic material 1 point: chiasmata is a visible result of crossing over

Describe Double Fertilization

1 sperm fuses with the egg to make a zygote, and the other fuses with a diploid cell to make a endosperm triploid.

Two mechanism of speciation

1. Allopatric speciation: geographic isolation; when two populations are are physically separated for too long and the two population eventually leads to reproductive isolation. 2. Sympatric speciation: reproductive isolation within population range (polyploidy) 3. Prezygotic barriers: reproduction barrier, mechanical isolation, temporal isolation 4. Postzygotic barriers: reduced hybrid viability, reduced hybrid, and hybrid breakdown

Two types of evidence to determine phylogeny of organism and strength of each type

1. Fossil record: observe past organisms (show direct evidence of common ancestors) 2. Homology- morphology: observation on the organisms structures and forms of vestigial structure (similarities in form shows common ancestry) 3. Homology- embryology: change in genes expression during development (similarities in development shows common ancestry) 4. Homology- reproduction: observation on reproductive strategies (similarities in reproduction shows common ancestry) 5. DNA sequence: comparison of DNA sequences in specific sequences (similarities in sequence show similarities in ancestry) 6. Biogeography: analysis of organismal distribution (use past and present information to show the similarity of ancestors) 7. Direct observation/behavior: observe organisms in natural habitat (similarity in habitat and behavior can show possible information about ancestry)

FRQ #8: a. Explain 3 processes using an appropriate example each b. Discuss the impact on biodiversity of each (increase or decrease

1. Mutation -Change in DNA Ex:/ Deletion/insertion point mutation chromosomal aberration Impact: Increase or decrease -Altered proteins -Raw material for selection 2. Population Bottlenecks -Sudden/drastic decrease in population size Ex:/ cheetahs -Decrease -Random/not adaptive -Not representative of smaller gene pool 3. Adaptive radiation -Multiple species from 1 ancestor Ex:/ Galapagos finches -Increase -New species

FRQ #9: Describe the different types of hereditary variability. Explain how this variability can lead to the origin and maintenance of species.

1. Mutation Origin -Sponaneous or induced -Mechanism of induction 2. Natural Selection -The fittest in relation survive Genetic Drift: Ranom change in gene frequenes

Two mechanism of evolutionary change

1. Sexual selection: certain phenotype will give mates an advantage over other "competitors" ; the traits that are advantageous will likely to be passed on. 2. Mutation: change in nucleotide sequence or amino acid sequence or protein structure. The ability for these changes to pass on requires the mutation to occur in the sex cells. Mutations can be good or bad.

enzyme in the process

1. lower activation energy and speed up reaction 2. synthesis: enzymes bring substrate closer together 3. digestion: active site binds substrate and puts stress on bonds that must be broken, making it easier to separate molecules

quantitative experiment on influence of pH

3 test tubes w 3 mL of h2o2, add 3 mL of either pH 4, 7, or 10, then set up computer with logger pro and gas pressure sensor. add 2 drops of enzyme into experiment and place stopper on lid which is connected to the pressure sensor and watch the graph for 3 min and find slope. the biggest slope is the highest rate, meaning that pH is optimal

The restriction site for an enzyme called Pvul is the following sequence: 5prime-CGATCG-3prime 3prime-GCTAGC-5prime One strand of a DNA molecule has the following sequence: 5prime-CTTGACGATCGTTACCG-3prime Draw the other strand. Will Pvul cut this molecule? If so, draw the products.

3prime-GAACTGGCTAGCAATGGC-5prime Yes, Pvu can cut it because GCTAGC is in it. The products would be GAACTG and AATGGC (on the other sides of the parts that are cut)

Owls (primary predators of rock pocket mice) are nocturnal visual predators that are able to distinguish effectively between dark and light colors in low-light conditions. Which of the following best explains the relationship between coat color and predation rate in the rock pocket mouse population on the lava flow rocks with respect to the different coat color phenotypes?

A dark coat color provides camouflage to mice on the lava flow rocks; therefore, mice with dark coats have a lower predation rate in that habitat.

Genetic disorders and diseases follow different inheritance patterns depending on the nature of the mutation involved, whether it is caused by an abnormal number of chromosomes or a mutation in the DNA itself AND the location of the gene on a particular chromosome. What are the various causes of genetic disorders and patterns of inheritance for disease genes?

A genetic disorder is a disease caused in whole or in part by a change in the DNA sequence away from the normal sequence. Genetic disorders can be caused by a mutation in one gene (monogenic disorder), by mutations in multiple genes (multifactorial inheritance disorder), by a combination of gene mutations and environmental factors, or by damage to chromosomes (changes in the number or structure of entire chromosomes, the structures that carry genes, or through codominant alleles like in sickle cell anemia by inheriting the dominant gene from both parents.

What are metabolic pathways?

A metabolic pathway is a linked series of chemical reactions occurring within a cell. The reactants, products, and intermediates of an enzymatic reaction are known as metabolites, which are modified by a sequence of chemical reactions catalyzed by enzymes.

What is entropy?

A thermodynamic quantity representing the unavailability of a system's thermal energy for conversion into mechanical work, often interpreted as the degree of disorder or randomness in the system.

1. 1994 Genetic variation is the raw material for evolution. a. Explain three cellular and/or molecular mechanisms that introduce variation into the gene pool of a plant or animal population. b. Explain the evolutionary mechanisms that can change the composition of the gene pool.

A) 1. Mutation: invents alleles that never before existed in the gene pool. 2. Sexual Reproduction: creates individuals with new combination of alleles. -- Random fertilization: each gamete is a unique set of recombined genes. During Sexual reproduction, Any of the numerous combinations of sperm could fuse with any of the eggs in the fallopian tube, thus creating a genetically unique zygote. 3.Chromosomal Mechanisms: Crossing over occurs during Prophase I of Meiosis exchanges homologous chromosomes and produces randomly recombined chromosomes. B) 1.Natural selection-natural process where organisms with certain genotypic characteristics make them able to survive and produce more offspring with similiar genotypes, eventually evolving the species as a whole in one that adapted over the course of generations to suit their environment. Outbreeding: organisms have no familial ties breed; shuffles alleles in gene pool; genetic variety. 2.Gene Flow: movement of genes through different populations of species; emmigration loses genetic diversity (less alleles to shuffle) and decreases gene pool, immigration increases genetic diversity and increases gene pool. 3. Assortative mating-common phenomenon; most mates are chosen nonrandomly; sexual selection= choose mate with best characteristics; or artificial selection= humans breed organisms for advantageous alleles.

3. 2001 Charles Darwin proposed that evolution by natural selection was the basis for the differences that he saw in similar organisms as he traveled and collected specimens in S. America and on the Galapagos islands. a. Explain the theory of evolution by natural selection as presented by Darwin. b. Each of the following relates to an aspect of evolution by natural selection. Explain three of the following. i. Convergent evolution and the similarities among species (ecological equivalents) in a particular biome (e.g., tundra, taiga, etc) ii. Natural selection and the formation of insecticide-resistant insects or antibiotic -resistant bacteria iii. Speciation and isolation iv. Natural selection and behavior such as kinesis, fixed-action-pattern, dominance hierarchy, etc v. Natural selection and heterozygote advantage

A)-reproductive potential- the ability to over produce -variability- inheritable changes or mutations linked to variability - limited resources- biotic or abiotic - competition- intraspecific struggle for existence -differential reproduction- reproductive success of variants - generations- time needed for evolution to occur B) ii.an inherited characteristic enables the organism to resist the effect of the toxin over time; HIV virus and constant evolvement iii.a single population divides into two reproductively isolated populations or equivalent (use of gene pools);role of barriers in speciation v.survival value of heterozygote over BOTH homozygotes; sickle-cell allele

7. Discuss the process of cell division in animals. A.Include a generalized description of the cell cycle B. Include a description of the chemical regulation system that governs the cell cycle both in terms of internal and external signals and the consequences of failure of this control system. C. Include a comparison of M phase activities if the cell were undergoing mitosis compared with meiosis.

A. Sloan: cell cycle is an ordered series of events involving cell growth and cell division that produces two new daughter cells. Cells on the path to cell division proceed through a series of precisely timed and carefully regulated stages of growth, DNA replication, and division that produces two identical (clone) cells. The cell cycle has two major phases: interphase and the mitotic phase. During interphase, the cell grows and DNA is replicated. During the mitotic phase, the replicated DNA and cytoplasmic contents are separated, and the cell divides. B. Cyclin-Cdk complexes can also be regulated by the binding of Cdk inhibitor proteins (CKIs). There are a variety of CKI proteins, and they are primarily employed in the control of G1 and S phase. 2)Cyclins: second set of proteins that control checkpoint switches have no enzymatic activity themselves but bind to kinases to allowing kinases to become active 3)APC anaphase promoting complex: activated once MCdk begins to form and it adds ubiquitin to the cyclin, which degrades the C. The cell separates the copied chromosomes to form two full sets (mitosis) and the cell divides into two new cells (cytokinesis). Only if the cell were undergoing meiosis, it would also include synapsis and crossing over, tetrads lining up on the metaphase plate, and the sister chromatids would stay attached at anaphase 1.

2010 Homeostatic maintenance of optimal blood glucose levels has been intensively studied in vertebrate organisms. A. Pancreatic hormones regulate blood glucose levels. IDENTIFY two pancreatic hormones and describe the effect of each hormone on blood glucose levels.

A. Insulin, lowers blood glucose levels Glucagon, raises blood glucose levels

12. The cell cycle is fundamental to the reproduction of eukaryotic cells. A. Describe the phases of the cell cycle. B.Explain the role of THREE of the following in mitosis or cytokinesis: • Kinetochores • Microtubules • Motor proteins • Actin filaments C. Describe how the cell cycle is regulated and discuss ONE consequence of abnormal regulation.

A. Interphase->prophase->prometaphase->metaphase->anaphase->telophase->cytokinesis OR G1->S->G2->M-> B. Kinetochores: Located in centromeres of condensed chromosomes and is the microtubule attachment site necessary for chromosome positioning and movement. Microtubules: Fundamental structural element of the spindle apparatus that provides the framework on which chromosomes motility is generated and defines axis of division and cytokinesis. Motor proteins: In kinetochores, motor proteins move chromosomes during mitosis, including anaphase separation. Motor proteins includes the usage of kinesins and dyneins. C. The cell cycle is regulated by checkpoints, which block cell cycle progress unless specific molecular conditions are satisfied. Checkpoint regulation is monitored by the action of MPF and CDKs, whose concentration determines if the cell will move on in the cell cycle. Also, hormones regulate the cell cycle as growth factors control cell cycle activity. If abnormal regulation occurs, it could lead to cancer due to uncontrolled cell proliferation and apoptosis.

2011-B The cell cycle is fundamental to the reproduction of eukaryotic cells. A. Describe the phases of the cell cycle.

A. Interphase-Prophase-(Prometaphase)-Metaphase-Anaphase-Telophase-Cytokinesis I: (G1, S, G2) chromatin dispersed in nucleus, DNA replication P: chromosomes condense PM: nuclear envelope disappears M: chromosomes align on metaphase plate A: chromosomes move to opposite poles T: nuclear envelope reassembles C: cell division

4. Sexual reproduction is used by many organisms as a mechanism to increase genetic variation. A. Identify and describe how meiosis helps contribute genetic variation in sexually reproducing organisms. B. Describe how meiosis accomplishes a reduction in chromosome number. C. Contrast the major differences between the daughter cells in meiosis and the daughter cells in mitosis.

A. Meiosis contributes to genetic variation using three mechanisms: it implements crossing over, which changes the gene sequences on individual chromosomes during prophase. It also uses independent assortment, where homologous chromosomes separate randomly and thus the gametes can receive any combination of unlinked alleles from either parent. Meiosis also creates many gametes, and thus when only 1 results in fertilization, random fertilization contributes to genetic variation by having only certain random gametes pass on their genetic information. B. Meiosis reduces the number of chromosomes by sending homologous chromosomes to opposite poles of the cell to form separate cells. Thus, meiosis 1 results in cells with chromosomes from only 1 parent(except the lengths of DNA that crossed over) but with 2 sister chromatids in each of these chromosomes. These sister chromatids separate in Meiosis II, which is similar to mitosis, so the resulting gametes have genetic material from 1 parent and only 1 copy of this genetic material. So that each resulting daughter cell has only half the amount of the parenting cell. C. Meiosis produces four haploid cells. Mitosis produces two diploid cells. The difference in the daughter cells is that the two mitosis produced are each diploid and regular while the four produced in meiosis are all haploid and used in sexual reproduction The daughter cells in mitosis are identical to the parent cell, while the daughter cells in meiosis are genetically unique.

2003-B A difference between prokaryotes and eukaryotes is seen in the organization of their genetic material. A. Discuss the organization of the genetic material in prokaryotes and eukaryotes.

A. P:E No introns; introns not in nucleus; nucleus/mito/chlor circular; linear no histones; histones

3. Mitosis occurs in many different cell types. The amount of time spent in each stage of mitosis varies and can be roughly estimated by observing a population of cells that is actively dividing. A. Design an experiment to determine the amount of time cells spend in each stage of mitosis relative to time spent in interphase. Identify a relevant cell type for the experiment. B. Describe the results expected from this experiment. C. Draw a chart or graph that visually depicts the expected results.

A. Take a photo of the microscope of onion cells under a microscope, count the amount of cells in interphase and the amount in mitosis, from their calculate percentage of cells that were in mitosis by dividing number of cells in mitosis by the total and repeat with interphase to compare relative time a cell spends in interphase and mitosis. For each stage of mitosis follow procedure from before except instead determine which phase each mitotic cell is in and record that data as well. B. Roughly less less than 10% of cells in mitosis and over 90%in interphase unless the cells are cancerous and the checkpoints aren't working/ there's a mutation of some sort invalidating the data. C. #of cells in mitosis: 95- 8.5% # of cells in interphase: 1023- 91.5% # of cells total: 1118- 100%

8. An organism is heterozygous at two genetic loci on different chromosomes. A. Explain how these alleles are transmitted by the process of mitosis to daughter cells. B. Explain how these alleles are distributed by the process of meiosis to gametes. C. Explain how the behavior of these two pairs of homologous chromosomes during meiosis provides the physical basis for Mendel's two laws of inheritance. D. Explain how nondisjunction could affect the resulting daughter cells.

A. replication occurred during interphase/S phase of cell cycle forming 2 identical chromatids. Prophase: nuclear envelope disappears; mitotic spindle begins to form; chromatin fibers become more tightly coiled, condemning into discrete chromosomes observable with a light microscope. During metaphase, chromosomes align at the center of the cell; unpaired chromosomes attach at spindle fibers (microtubules) independently of each other; During anaphase, the spindle fibers shorten and the chromatids separate; Through cytokinesis,the non-kinetochore spindle fibers elongate and form a cleavage furrow or a cell plate may form (compartmentalization) resulting in 2 genetically identical daughter cells; each daughter nucleus contains A, a, B, b; all alleles are still present in each. B. Homologous chromosomes (maternal/paternal) pair (at synapse) during prophase I; this pairing forms tetrads. Crossing over occurs at chiasmata. During metaphase tetrads line up on metaphase plate; during anaphase I, chromosomes move toward poles; sister chromatids remain attached at centromere; **homologous chromosomes, each composed of two sister chromatids, move toward opposite poles. After meiosis I is finished, there is no second replication and daughter cells have half the genetic material of parent cell, thus they are haploids. Meiosis II is like mitosis; during anaphase II, chromatids that were lined up during metaphase I separate (two sister chromatids of each chromosome are not genetically identical due to crossing over); There are two cytokinetic events resulting in 4 haploid cells/gametes. Each of the four daughter cells is genetically different from other daughter cells and from the parent cell C. Through independent assortment, the alleles separate into gametes independent of each other. Independent assortment is also due to random alignment of tetrads (homologous pairs) during metaphase I. Law of segregation states that allele pairs (homologues) separate during anaphase I gamete formation and randomly unite during fertilization; each gamete receives paternal or maternal allele. Linked alleles do not show independent assortment D. Failure of homologs to separate in Anaphase I (resulting in four affected daughter cells). Failure of sister chromatids to separate in Anaphase II (resulting in only two daughter cells being affected)

1. Two Drosophila heterozygous for the same set of alleles were crossed: A+A x A+A. 65% of the resulting offspring had the mutant phenotype associated with the A allele, and 35% had the wild type phenotype. Determine if the homozygous mutant genotype, AA, is more like to be lethal or to result in the mutant genotype. A. Show the expected genotypic results of the cross. B. Show the expected phenotypic results of the cross. C. Explain the phenotypic ratio for a dominant lethal allele.

A. ¼ of the offspring would be homozygous for the wild type allele, ½ of the offspring will be heterozygous, and ¼ of the offspring will be homozygous for the mutant genotype. B. ¾ of the offspring will have the mutant phenotype, and ¼ of the offspring will have the wild type phenotype. C. Since it is a dominant allele, it is expressed in both homozygotes and heterozygotes. In this case, that is around 75% of the population. Since it is lethal, that means that around 75% of the population will die from it. 3:1

How do carbohydrate metabolic pathways, glycolysis, and the citric acid cycle interrelate with protein and lipid metabolism pathways?

ATP produced by catabolic pathways is used to drive anabolic pathways. many of the intermediated of glycolysis and the CAC are used in the biosynthesis of a cell's molecule.

Structural changes that can occur to a protein after translation to make it function properly

Activation into a functional protein through cleavage of certain amino acid sequences; the amino acid sequence can fold to form the secondary or tertiary structure

explain the difference between active and passive transport

Active transport requires energy, whereas passive transport does not

3. Fermentation is an alternate method of producing energy in many organisms and cell types. Contrast the advantages and disadvantages of aerobic and anaerobic metabolism.

Advantages: -Oxygen is not needed, so it is not reliant on oxygen from the body Disadvantages: -Not the same amount of ATP being produced

How does atomic structure determine the properties of elements, molecules, and matter?

An atom is the smallest unit of matter that retains all of the chemical properties of an element. The attraction between the positively charged protons and negatively charged electrons holds the atom together. Most atoms contain all three of these types of subatomic particles—protons, electrons, and neutrons.

How do electrons move through the electron transport chain and what happens to their energy levels?

As electrons move through the electron transport chain, they go from a higher to a lower energy level and are ultimately passed to oxygen (forming water). Energy released in the electron transport chain is captured as a proton gradient, which powers production of ATP by a membrane protein called ATP synthase.

(b) Explain why DNA replication cannot proceed to the regions of the chromosome labeled as I in Figure 1B unless topoisomerase binds ahead of each advancing replication fork in region II.

Because replication only follows single strands, not double strands, of DNA. As you can see in the figure, the regions of the chromosome marked as I are on a double strand of DNA and replication proceeds only in regions of single strands of DNA. Topoisomerase, is the enzyme responsible for binding the double strand of DNA and separating each strand, forming two single strands, before the replication bubble arrives, only then can replication continue. For this reason, we can say that in the region of the chromosome marked as I, it will be necessary for the topoisomer to separate each strand of DNA for replication to proceed.

What are the role of acids, bases, and buffers in dynamic homeostasis?

Buffers readily absorb excess H+ or OH-, keeping the pH of the body carefully maintained in the narrow range required for survival. ... The buffer maintaining the pH of human blood involves carbonic acid (H2CO3), bicarbonate ion (HCO3-), and carbon dioxide (CO2).

Figure 1. Models of a prokaryote, first eukaryotic common ancestor, and last eukaryotic common ancestor. Based on the information provided in the figure, which of the following best describes how membrane-bound organelles provided unique evolutionary advantages to the last eukaryotic common ancestor?

Cells with mitochondria were able to utilize metabolic energy more efficiently.

What is the role of carbohydrates in cells and in the extracellular materials of animals and plants?

Carbohydrates are a group of macromolecules that are a vital energy source for the cell and provide structural support to plant cells, fungi, and all of the arthropods that include lobsters, crabs, shrimp, insects, and spiders.

. Relationships of selected organisms based on catalase structure Which of the following student claims is best supported by the data provided?

Cattle and sheep are separated by only one node, and they are separated from the other organisms by more than one node.

What is the role of cells in organisms?

Cells provide structure for the body, take in nutrients from food and carry out important functions. Cells group together to form tissues, which in turn group together to form organs, such as the heart and brain.

4. 2009 Phylogeny is the evolutionary history of a species. (a) The evolution of a species is dependent on changes in the genome of the species. Identify TWO mechanisms of genetic change, and explain how each affects genetic variation. (c) Describe TWO types of evidence—other than the comparison of proteins—that can be used to determine the phylogeny of organisms. Discuss one strength of each type of evidence you described.

Changes in DNA-mutations via insertions/deletions of dna sequences/codons; therefore changes phenotype in some way (Most mutations are unnoticable; others are lethal; few are advantageous) Changes in population-allopatric=diverge due to enviro; symm=new species evolve from common ancestor even though in same geographic region; producing reproductive fitness

How does cholesterol help maintain the fluid nature of the plasma membrane of cells?

Cholesterol functions as a buffer, preventing lower temperatures from inhibiting fluidity and preventing higher temperatures from increasing fluidity.

The unique properties (characteristics) of water make life possible on Earth. Identify three of those properties.

Cohesion: Water bonds with other water molecules. This allows for water to be pulled up through the xylem of plants. Adhesion: Water bonds to other surfaces. This also allows it to be pulled up through the xylem. Universal solvent: Water can be used to dissolve many other chemicals and substances.

2003-B B. Contrast the following activities in prokaryotes and eukaryotes: - Replication of DNA - Transcription or translation

DNA Rep: P;E single origin, multiple origin no telomeres, telomeres Translation: T+T coupled; T+T not coupled 30s/40s; 40s/60s

To explain the movement of DNA fragments in the comet assay, identify one property of DNA and provide reasoning to support how the property contributes to the movement during the comet assay technique.

DNA has a negative charge which contributes to movement because DNA moves toward the oppositely charged pole.

What are the differences among the different types of endocytosis: (phagocytosis, pinocytosis, and receptor-mediated endocytosis) and exocytosis?

Endocytosis takes particles into the cell that are too large to passively cross the cell membrane. Phagocytosis is the taking in of large food particles, while pinocytosis takes in liquid particles. Receptor-mediated endocytosis uses special receptor proteins to help carry large particles across the cell membrane.

How do chemical reactions play a role in energy transfer?

Energy plays a key role in chemical processes. According to the modern view of chemical reactions, bonds between atoms in the reactants must be broken, and the atoms or pieces of molecules are reassembled into products by forming new bonds. Energy is absorbed to break bonds, and energy is evolved as bonds are made.

In a few species, chloroplast genes are inherited only from sperm. How might this influence efforts to prevent transgene escape?

Engineering the transgene into the chloroplast DNA would not prevent its escape. Such a method requires that the chloroplast DNA be found only in the egg.

What are the four major types of lipids?

Fats and oils, waxes, phospholipids, and steroids.

What are functions of fats in living organisms?

Fats play a vital role in maintaining healthy skin and hair, insulating body organs against shock, maintaining body temperature, and promoting healthy cell function. Fat also serves as a useful buffer against a host of diseases.

What is the fundamental difference between anaerobic cellular respiration and the different types of fermentation?

Fermentation and anaerobic respiration differ because although they both start with glycolysis, fermentation does not stop with the products of glycolysis, but instead created Pyruvate and continues the same path as aerobic respiration.

How do functional groups determine the properties of biological molecules?

Functional groups are a set of commonly found groups of atoms covalently bonded to carbon in organic molecules. The unique functional groups present on a biomolecule determine reactivity, solubility, and other physical properties that lead to its biological role in living organisms.

What are the roles of tight junctions, gap junctions, and plasmodesmata in allowing cells to exchange materials with the environment and communicate with other cells?

Gap junctions in animal cells are like plasmodesmata in plant cells in that they are channels between adjacent cells that allow for the transport of ions, nutrients, and other substances that enable cells to communicate.

What is the overall result, in terms of molecules produced, in the breakdown of glucose by glycolysis?

Glycolysis begins with glucose and produces two pyruvate molecules, four new ATP molecules, and two molecules of NADH. (Note: two ATP molecules are used in the first half of the pathway to prepare the six-carbon ring for cleavage, so the cell has a net gain of two ATP molecules and two NADH molecules for its use).

Over the next year, a virus attacks and rapidly kills almost all plants of the tobacco species. A closely related species of tobacco that is resistant to the virus becomes established during that same year. The nectary tube in the resistant species averages 7.5cm. Scientists monitored the area for the next five years to determine how this change affected the moth population. Which of the following best describes the expected distribution of proboscis size of the moths at the end of the five‑year study?

Graph A.

A molecule of messenger RNA has just been synthesized in the nucleus of a human cell. What type of modifications may occur to this RNA before it leaves the nucleus?

Guanine attaches to the cap end and adenine attaches to the poly-a tail; introns are removed, leaving exons

b. Darwin's ideas have been enhanced and modified as new knowledge and technologies have become available. Discuss how TWO of the following have modified biologists' interpretation of Darwin's original contributions. o Hardy-Weinberg equilibrium o Punctuated equilibrium o Genetic engineering

Hardy-Weindberg-under certain conditions, no evolution occurs; equation indicates gradual change over time; p+q=1 P squared blech and explain variables; conditions MUST BE MET 1) large pop size/ no drift 2) No emm/immigration 3) No net mutations 4)No sexual selection-random mating 5)No natural selection Hardy-Weinberg is null in determining cause for change, only calculates it. Punctuated equilibirum-long periods of stasis then sudden change; possible rapid change by cladogenesis (only divergent evolution, species does not evolve) Draw pictar of model

Predict the likely results of a comet assay for this treatment.

Head (only) OR (head with) no tail

Characteristics of a Fern

Hermaphroditic, produces egg and sperm, Homospory

Characteristics of a Flower

Heterospory, pollen, made of tube cells, and 2 sperm, ovule, large diploid, and egg cell

Mechanism that prevent self-fertilization

Higher stigma than anther

Explain why the arrangement of water molecules is different in ice and water.

Hydrogen bonding plays a stronger role in the density and shape of ice than the covalent bonds as the water molecules are pressed against each other very closely. This causes like-like charges to repel and spread out, making ice less dense than water.

Describe why hydrogen bonds form between water molecules.

Hydrogen bonds form between a hydrogen atom and a very electronegative atom. Since oxygen is very electronegative, hydrogen bonds form between hydrogen and oxygen.

(c) Identify the plasmid that must contain the CD3γ core promoter sequence but the fewest or no negative regulatory sequences. Based on the data in Figure 2, describe the most likely cause of the variation in luciferase activity among the cells that contain plasmids pCD3γ-419, pCD3γ-309, pCD3γ-239, and pCD3γ-199. Calculate the approximate percent increase in luciferase activity between cells containing plasmid pCD3γ-59 and cells containing plasmid pCD3γ-149. Round to the nearest whole number.

I HAVE NO IDEA

Ethical considerations aside, if DNA-based technologies became widely used, discuss how they might change the way evolution proceeds, as compared with the natural evolutionary mechanisms that have operated for the past 4 billion years.

If DNA-based technologies became widely used, this could change the way evolution proceeds compared to 4-billion years ago because now we can see and read all diseases that people have. Knowing that a baby has a disease when they are born, and knowing what parent or parents it came from makes it easier to avoid spreading this disease in the future instead of having it spread across from generation to generation. 4-billion years ago we didn't know anything about where these diseases where coming from or what they even were for that matter. This led to low survival rates because they was no information on the diseases, so no one knew what to do about them. This also led to people dying at younger ages, and not being able to withstand the seriousness of the diseases that they have.

What are the differences among ionic bonds, covalent bonds, polar covalent bonds, and hydrogen bonds?

In an ionic bond, an atom gives away one or more electrons to another atom. In a covalent bond, two atoms share one or more electrons. A hydrogen bond is a relatively weak bond between two oppositely charged sides of two or more molecules.

Why does the term "carbon fixation" describe the products of the Calvin cycle?

In carbon fixation, CO2 is attached to RuBP by the enzymes. it results in 6-carbon products which splits into two even 3-carbon compounds.

2010 B. For ONE of the hormones you identified in (a), IDENTIFY one target cell and discuss the mechanism by which the hormone can alter activity in that target cell. Include in your discussion a description of reception, cellular transduction, and response.

Insulin Target cells: any cells except red blood cells/brain cells Reception: ligand binding to two adjacent monomers forms an active dymer Transduction: stimulates a cascade pathway/mediated by a second messenger/amplifies signal Response: raises cellular uptake of glucose.

How does the molecular structure of water result in unique properties of water that are critical to maintaining life?

It is polar, allowing for the formation of hydrogen bonds, which allow ions and other polar molecules to dissolve in water. ... The hydrogen bonds between water molecules give water the ability to hold heat better than many other substances.

(d) Predict the most likely observed level of luciferase activity if plasmid pCDγ3-789 is introduced into nonlymphoid cells such as a line of kidney tissue cells. Provide reasoning tojustify your prediction.

Its level will be high because introducing plasmid into non lymphoid kidney tissue cell will most likely bring about blockages to the kidney tissue cell and its activities. Hence this is likely going to trigger the presence of luciferase to continually disintegrate the plasmid substances in order to promote smote flow of soluble substance's across the body

What increases transpiration rates

Larger leaves

What are the reactions in the Calvin cycle described as the light-independent reactions?

Light reactions harness energy from the sun and produce chemical bonds, ATP and NADPH. They require light.

Embryo Protection in Ferns

Limited, retained in female gametophyte tissue

Why is carbon important for life?

Living things need carbon the most in order to live, grow, and reproduce. Also, carbon is a finite resource that cycles through the Earth in many forms. Also, carbon is so important to life because virtually all molecules in the body contain carbon.

In general, what are the effects of histone acetylation and DNA methylation on gene expression?

Methyl groups condense chromatin which is associated with a lack of gene expression. Histone acetylation loosens chromatin structure and is associated with gene expression.

How are monosaccharide building blocks assembled into disaccharides and complex polysaccharides?

Monosaccharides are converted into disaccharides in the cell by condensation reactions. Further condensation reactions result in the formation of polysaccharides. These are giant molecules which, importantly, are too big to escape from the cell.

Mollusk fossils were excavated and collected from a coastal cliff. Scientists studying the fossils observed patterns of changing shell shape in the mollusks. The scientists identified a period of stability or stasis, followed by rapid change, and then stasis again with regard to shell shape. Which of the following statements best predicts the result of a comparison between the fossils found before and after the period of rapid change?

Most of the species found in the first static period will be absent in the second static period.

How do plants absorb energy from sunlight?

Most plants contain chlorophyll that absorbs the sun's energy and turns it into chemical energy.

What are the main cellular structures involved in photosynthesis?

Multicellular autotrophs use chloroplasts, thylakoids, and chloroplasts.

Students created a model showing one possible mechanism for speciation in which a geographic barrier separated one large population into two smaller populations, A and B. Over time, populations A and B developed into two distinct species. The model is shown in Figure 1. Which of the following best describes what occurred during the periods indicated by the arrows?

Natural selection acted on variation that was present in both populations A and B, and the lack of gene flow eventually led to the formation of two new species.

A few companies in China and South Korea provide the serving of cloning dof, using cells from their client's pets to provide nuclei in procedures. Should their clients expect the clone to look identical to their original pet? Why or why not? What ethical question does this bring up?

No because the DNA would be the same, but may not be equally expressed. Differences in environment also contribute. In order to get 1 Dolly that survived, many Dollys were created and died. Is it worth it for one animal to live when many are killed in the process?

2004 Darwin is considered the "father of evolutionary biology." Four of his contributions to the field of evolutionary biology are listed below. o The nonconstancy of species o Branching evolution, which implies the common descent of all species o Occurrence of gradual changes in species o Natural selection as the mechanism for evolution a. For EACH of the four contributions listed above, discuss one example of supporting evidence.

Nonconstancy- species are variant in populations; change in species over course of evolution; change in pop numbers over time; Ex. Dogs. (Can be bred, but considered different species = very different offspring) Branching Evolution-homologous structures with skeletons in human arms, cat limbs and whale limbs; everything from common ancestor; divergent evolution produced "branching" species Gradual changes-genes mutate=selection occurs= population evolves giraffe necks Evolution-survivors pass genes onto next generation; antibiotics and resistant bacteria

What are the steps of the scientific method?

Observation, Question, hypothesis that answers a question, Prediction based on the hypothesis, Do an experiment to test the prediction, Analyze the results

What are the different classifications of carbohydrates?

On the basis of the number of forming units, three major classes of carbohydrates can be defined: monosaccharides, oligosaccharides and polysaccharides.

Membranes are essential components of all cells. Identify TWO macromolecules that are components of the plasma membrane in a eukaryotic cell and discuss the structure of each.

One structure of the plasma membrane is carbohydrates. Carbs can be monomers, chains of monomers, or branches. They provide structure for the membrane Another structure is phospholipids. They contain a hydrophilic head and a hydrophobic tail. They move with great fluidity, allowing materials to pass in and out of the membrane

Describe the levels of organization among living things.

Organelles, cells, tissues, organs, organ systems, organisms, populations, communities, ecosystems, the biosphere.

Identify and describe the properties of life.

Organized structure, respond to stimuli, reproduction, growth, regulation- nutrient transport, homeostasis- constant internal conditions, energy Processing, adaptation, evolution.

What mechanisms control cellular respiration?

Pathways are regulated through enzymes that catalyze individual steps of the pathway. ATP, ADP, and NADH are molecules that regulate cellular respiration enzymes.

Describe THREE types of chemical bonds/interactions found in proteins. For each type, describe its role in determining protein structure.

Peptide bonds, hydrogen bonds, and disulfide bridges are all found in proteins.Peptide bonds link amino acids in a protein's primary structure. Hydrogen bonds hold together the secondary structure (the alpha helices or beta pleated sheets). Disulfide bridges stabilize the tertiary structure by allowing different parts of the protein to be held together covalently.

What is the molecular structure of phospholipids, and what is the role of phospholipids in cells?

Phospholipids are a class of lipids that are a major component of all cell membranes. They can form lipid bilayers because of their amphiphilic characteristic. The structure of the phospholipid molecule generally consists of two hydrophobic fatty acid "tails" and a hydrophilic "head" consisting of a phosphate group.

Identify THREE macromolecules that are components of the plasma membrane in a eukaryotic cell and discuss the structure and function of each

Phospholipids, cholesterol, and proteins are components of the plasma membrane in a eukaryotic cell. Phospholipids are made up of a glycerol, two fatty acids, and a polar head group with phosphate. Phospholipids are selectively permeable, protecting the cell. Cholesterol is a ring structure that stabilizes the membrane. Proteins are made up of amino acids, and act as enzymes to speed up chemical reactions.

What is the role of photosynthesis in the energy cycle of all living organisms?

Photosynthesis captured light and allows autotrophs to make their own food/ energy.

What is the relevance of photosynthesis to living organisms?

Photosynthesis is the number one source of oxygen in the atmosphere and provides food from sunlight, carbon dioxide, and water. it's the primary source of energy.

How and where does photosynthesis occur within a plant?

Photosynthesis occurs in the chloroplasts, and uses the chlorophyll to fully make photosynthesis happen.

Organization of the genetic material in prokaryotes and eukaryotes

Prokaryotes: DNA is a single loop that is not bound within a nucleus because they lack a nucleus Eukaryotes: DNA is organized in chromosome which are contained in a membrane-bound nucleus

Gene regulation in prokaryotes and eukaryotes

Prokaryotes: control of gene expression is mostly at the transcriptional level Eukaryotes: regulation of a gene expression can occur at all stages of the transcription and translation process

Transcription/translation in prokaryotes and eukaryotes

Prokaryotes: mRNA produced by transcription is immediately translated without additional processing Eukaryotes: pre-mRNA is processed in various ways before leaving the nucleus as mRNA

Replication of DNA in prokatyotes and eukaryotes

Prokaryotes: occurs in the cytoplasm; one origin per DNA molecule; rapid process Eukaryotes: occurs in the nucleus; many origins of replication; slow process

Cell division in prokaryotes and eukaryotes

Prokaryotes: undergoes binary fission Eukaryotes: undergoes mitosis

What is the relationship between protein shape and function?

Proteins are composed of amino acid subunits that form polypeptide chains. Enzymes catalyze biochemical reactions by speeding up chemical reactions, and can either break down their substrate or build larger molecules from their substrate.

Based on the chemical structure of cyanide, identify ONE type of biological macromolecule that could serve as a chemical precursor for the production of cyanide in a plant. Justify your choice.

Proteins could serve as a chemical precursor for the production of cyanide in a plant because proteins contain nitrogen, which can then triple bond with carbon in the plant, resulting in cyanide.

What are the functions of the major cell structures?

Provides storage and work areas for the cell; the work and storage elements of the cell, called organelles, are the ribosomes, endoplasmic reticulum, Golgi apparatus, mitochondria, lysosomes, and centrioles. Make enzymes and other proteins; nicknamed "protein factories".

Question 2

Researchers studying the phylogenetic relationships among African elephants (Loxodonta africana), Asian elephants (Elephas maximus), and woolly mammoths (Mammuthus primigenius) analyzed cytochrome b DNADNA sequences from several organisms of each species. Cytochrome b is a mitochondrial protein that functions in the electron transport chain. Partial sequences of cytochrome b mitochondrial DNADNA are often used in phylogenetic analyses. DNADNA was obtained from mammoth bone fragments radiocarbon dated to between 12,000 and 14,000 years ago and from living African and Asian elephants. Table 1 shows a partial cytochrome b DNADNA sequence for the reference individual (an African elephant) and some of the sample individuals studied. Dugongs (Dugong dugon) were identified as the outgroup in this study. Dugongs are marine mammals that are relatives of the larger group of elephant-like animals, the proboscideans.

A molecule of messenger RNA has just been synthesized in the nucleus of a human cell. Once in the cytoplasm, how is the mRNA translated to a protein?

Ribosomes attach to the start of the mRNA and brings over the necessary amino acids which form a peptide chain

What is the difference between saturated and unsaturated fatty acids?

Saturated fatty acids lack double bonds between the individual carbon atoms, while in unsaturated fatty acids there is at least one double bond in the fatty acid chain.

Question 1

Scientists interested in the relationship among vision, foraging for food, and fitness studied three isolated populations of a small species of primate whose diet includes small insects and fruit, both of which the animals primarily identify by sight. The three populations were once part of a larger population that was fragmented as a result of habitat destruction by humans. The study populations were selected on the basis of the significantly different vegetation among their current habitats. The scientists analyzed the frequency in each population of each of three alleles of a gene encoding an opsin protein. Opsin proteins are important for color vision, and each allele provides maximum sensitivity to a specific wavelength of visible light and thus to certain colors. Analysis of the alleles present in each population was performed in two different ways, and the data were combined to calculate the frequencies (Table 1).

13. Discuss Mendel's laws of segregation and independent assortment. Explain how the events of meiosis I account for the observations that led Mendel to formulate these laws.

Segregation is when you take all the people and separate them into white people and everyone else. Well same for alleles except more like sexism than racism cause the one from Mom and the one from your dad separate. So if your mom gave you a general for blue eyes and dad for green then your child has a 5050 chance of getting either allele. In anaphase 1 the tetrads of the duplicated homologs separate and you get haploid dinguses. Independent assortment is when you can line them up on either side. So the chromosomes split into the different cells independent of the other chromosomes. In metaphase they line up randomly along the plate.

What are the differences between short and long wavelengths of light? What wavelengths are used in photosynthesis?

Short wavelengths are high energy, and long wavelengths are low energy radio waves.

Describe why translation of an mRNA can start before transcription of the mRNA is complete in prokaryotes but not in eukaryotes.

Since mRNA is produced in the cytoplasm in prokaryotes, the start codons of an mRNA being transcribed are available to ribosomes before the entire mRNA molecule is made. In eukaryotic cells, transcription takes place in the nucleus. The mRNA must be completely synthesized and moved through the nuclear membrane to the cytoplasm before translation can begin.

In a particular region, a species of lizard lives in the lowlands. Even though there are numerous mountains in the region, the climate on the mountains is too cold for the lizards to survive. Scientists are concerned that the lowlands will become much warmer by the end of the twenty-first century and will become unsuitable for this lowland lizard species. Which of the following best predicts the impact on the lizard species if there is a gradual increase in the average temperatures found in both the lowlands and mountains in this region?

Some lizards will migrate to new habitats in the mountains and become isolated, leading to allopatric speciation.

Specimen 1 Specimen 2 Specimen 3 Specimen 4 Amino acid differences* 5 21 18 11 Percent of original carbon-14 remaining 38 73 25 11 Composition Sandstone Sandstone Limestone Shale DNA analysis 35% Guanine 28% Guanine 12% Guanine 29% Guanine *amino acid differences of a conserved protein compared with a fifth fossil specimen Which of the specimens indicated in the table is the oldest?

Specimen 4

(d) Referring to Figure 1B, explain why any newly synthesized strand of DNA is the result of both continuous and discontinuous DNA synthesis.

The DNA polymerase is the enzyme which copies in the direction of 5' to 3' in the leading strand and in case of lagging stand it is opposite. that is from the 3' to 5'. Replication in the 3' to 5' takes place in the fragments which is laid by the Okazaki fragments. These small fragments are kept together by the help of the enzyme known as DNA Ligase. So, called as discontinues. Incase of leading strand, the strand is in the same direction of the for the process of replication (5' to 3'). So it is continuous. Hence, the whole process is a combination of continuous and discontinuous DNA synthesis.

The degrees of relatedness in several native populations of prairie chickens in Marion and Jasper Counties in Illinois were determined. The degree of relatedness is inversely proportional to genetic diversity; that is, populations with a high degree of relatedness have less genetic diversity. Table 1 shows the collected data. Populations J1 through J5 are located in Jasper County. Populations M1 through M5 are located in Marion County. Numbers above 0.125 indicate a high degree of relatedness. Table 1. RELATEDNESS OF PRAIRIE CHICKEN POPULATIONS Population Degree of Relatedness J1 0.215 J2 0.215 J3 0.339 J4 0.280 J5 0.362 M1 0.305 M2 0.365 M3 0.199 M4 0.432 M5 0.405 Scientists claim that some of the populations in Marion County are at an increased risk of decline or extinction. Based on the data in table 1, which of the following best provides evidence to support this claim?

The M4 and M5 populations have the lowest genetic diversity of all populations.

The pH of a solution determines the charge of certain R groups. The pH of pineapple fruit ranges from 3.5 to 5.2. Predict and justify the effect on the activity of bromelain if it is used in a product with a pH of 11.

The activity of bromelain will decrease. The stems and fruits of pineapple plants, with a pH ranging from 3.5 to 5.3, contain bromelain. Since bromelain functions in this acidic environment, the activity of bromelain would decrease in a very basic environment.

2. Mitosis and meiosis have specific functions in different types of organisms. Discuss the advantages and disadvantages of sexual reproduction as compared to asexual reproduction.

The advantages of sexual reproduction include increasing genetic variability through things like crossing over and independent assortment, but it requires a mate and normally takes longer and energy release. Asexual reproduction can happen much more quickly and it only requires one organism, but it produces identical copies with little genetic variation.

(b) Identify the independent variable in the experiment described. Identify the plasmid that was used as a negative control for luciferase activity. Justify including the plasmid with the non-CD3γ active promoter in the experiments.

The aim of the experiment is to analyse regulatory sequences present upstream of the CD3γ gene that might influence the expression of the gene. To do this, they took a plasmid with a luciferase gene present. They then inserted sequences of varying length in the plasmid upstream of the luciferase genes. They then inserted this plasmid into T cells and performed luciferase assays to assess how highly expressed the luciferase gene was from the different regulatory sequences. This would be a good indication of the regulatory activity of that sequence. The only variable that changes throughout the experiment is the regulatory sequence that is inserted upstream of the luciferase gene. This means researchers can determine the effect of that regulatory sequence on the expression of a gene, without the influence of other factors. The plasmid used as a negative control is one that we expect not produce any luciferase activity. In this case, it is the parent plasmid, which does not have an upstream promoter/regulatory sequence, so it will not be able to be expressed, and the assay will detect no (or very little) luciferase activity. This is shown in the attached figure 2. This controls for expression of luciferase not produced by the promoter. The plasmid with the non-CD3 active promoter shows the "normal" activity of the luciferase gene. This controls for the possibility that the inserted sequences do not positively affect gene expression. If this happened and we did not have our control, we wouldn't know if the assay is working properly or whether this sequence just doesn't act as a promoter. Therefore, the non-CD3 active promoter shows us that the luciferase assay is working as expected, and we can therefore judge any changes in expression caused by our regulatory sequence.

The island was surveyed again in 2015, and scientists determined that the giant kangaroo rats originally in population A are now to be considered as a new species. Which of the following best explains the mechanism that caused this speciation event in the giant kangaroo rats of population A?

The animals in population A were reproductively isolated from those in populations B, C, and D.

Describe how the properties of water contribute to transpiration and thermoregulation in endotherms

The cohesive properties of water allow water to be pulled upward in narrow spaces. This allows for transpiration, through which water moves up the plant and evaporates at the surface of the leaf cells. Water has a very high heat capacity, which allows for thermoregulation in endotherms, since evaporation of water in sweat removes heat and helps to cool body temperature in endotherms.

How do the various components of the cytoskeleton perform their functions?

The cytoskeleton gives a cell its shape, offers support, and facilitates movement through three main components: microfilaments, intermediate filaments, and microtubules.

What is the difference between dehydration (or condensation) and hydrolysis reactions?

The difference between dehydration synthesis and hydrolysis is that in one, bonds are being formed, while in the other bonds are being destroyed. Dehydration synthesis bonds molecules together by removing water. In hydrolysis, water is added to the molecules in order to dissolve those bonds.

What is Rf equal to?

The distance travelled by the component divided by the distance travelled by the solvent.

What is the relationship between the structure and function of the components of the endomembrane system, especially with regard to the synthesis of proteins?

The endomembrane system processes and ships proteins specified by the nucleus. In the nucleus, DNA is used to make RNA, which exits the nucleus and enters the cytoplasm of the cell. The ribosomes on the rough ER use the RNA to create the different types of protein needed by the body.

How is ATP used by the cell as an energy source?

The enzymatic removal of a phosphate group from ATP to form ADP releases a huge amount of energy which is used by the cell in several metabolic processes as well as in the synthesis of macromolecules such as proteins.

What are the components of the extracellular matrix?

The extracellular matrix has three major components: Highly viscous proteoglycans (heparan sulfate, keratan sulfate, chondroitin sulfate), which cushion cells. Insoluble collagen fibers, which provide strength and resilience.

What is the difference between the first and second laws of thermodynamics?

The first and second law are the most frequently used laws in thermodynamics. The first law says that energy can be neither created nor destroyed. The first law is simply another version of the law of conservation of energy. The second law, on the other hand, asserts that some thermodynamic processes are forbidden.

How does the fluid mosaic model describe the structure and components of the plasma cell membrane?

The fluid mosaic model describes the structure of the plasma membrane as a mosaic of components —including phospholipids, cholesterol, proteins, and carbohydrates—that gives the membrane a fluid character.

What is the importance of electrons for the transfer of energy in living systems?

The transfer of electrons between molecules via oxidation and reduction is important because most of the energy stored in atoms is in the form of high-energy electrons; it is this energy that is used to fuel cellular functions.

Explain how the folding of the mitochondrial inner membrane and the many individual chloroplast thylakoids are beneficial to a cell that is carrying out chemical reactions.

The folding of the membrane increases the surface area, as does the thylakoids. With more surface area, more reactions are able to take place.

What is free energy, and how does free energy related to activation energy?

The free energy of a system changes during energy transfers, such as chemical reactions, and this change is referred to as ΔG or Gibbs free energy. The ΔG of a reaction can be negative or positive, depending on whether the reaction releases energy (exergonic) or requires energy input (endergonic).

The researchers built an enclosure to prevent migration into and out of the O'Neill rock pocket mouse population for a period of two years. Which of the following best predicts the effect of the enclosure on the O'Neill population?

The frequency of the MC1RD allele will decline due to natural selection.

What is the cell theory?

The historic scientific theory, now universally accepted, that living organisms are made up of cells, that they are the basic structural/organizational unit of all organisms, and that all cells come from pre-existing cells.

A famous study from the nineteenth century involves the peppered moth, Biston betularia, commonly found in Great Britain and Ireland. At that time, the moths, normally white with black speckles, known as the peppered variety, became less common than the solid black variety. The population was predominantly black throughout the nineteenth century. By the end of the twentieth century, the peppered version of the moth was again more common than the black version. Which of the following best explains the phenotypic pattern in the moths over time?

The increased air pollution of the nineteenth century due to the use of coal led to a darkening of surfaces from soot. The darker backgrounds conferred a selective advantage to the black variety, as described by natural selection. A shift to less polluting fuel in the twentieth led to cleaner air and a lightening of the moth's normal habitat.

Describe the importance of the inner membrane separating different regions of the mitochondrion and the chloroplast.

The inner membranes allow the organelles to compartmentalize because of increased folds, therefore, increasing surface area and efficency.

Identify the most likely cellular location of a mutant CFTR protein that has an amino acid substitution in the ATP-binding site.

The location would be the plasma membrane because the mutant protein is a gated ion channel on the membrane.

(a) Identify both the cellular component and the location of the component that is responsible for producing the luciferase protein from mRNAs transcribed in the plasmid-containing T lymphocytes. Explain what dictates to the lymphocytes the correct order in which amino acids should be linked to form the luciferase protein.

The mRNA which is read by the ribosome dictates the correct arrangement of amino acids through particular tRNA. The luciferase mRNA is moved to its similar protein in the ribosome which is seen in the cytoplasm of the cell. The chain of ribosome sits on the mRNA and forms a structure In the cytoplasm, called a polysome. this is useful in the fast synthesis of protein. The three-nucleotide genetic code in the mRNA which is being studied by the ribosome orders the right placement of amino acids through particular tRNA.

What are structural differences between animal and plant cells?

The main structural differences between plant and animal cells lie in a few additional structures found in plant cells. These structures include: chloroplasts, the cell wall, and vacuoles.

Discuss the organization of the genetic material in prokaryotes and eukaryotes.

The majority of genetic material is organized into chromosomes. Prokaryotes have a single circular chromosome in the nucleoid, whereas eukaryotes have multiple linear chromosomes in the nucleus.

a major distinction between prokaryotes and eukayotes is the presence of membrane-bound organelles in eukaryotes. Describe the structure and function of TWO eukaryotic membrane-bound organelles other than the nucleus or the plasma membrane.

The mitochondria has an inner and outer membrane and the inside has many folds. It plays a role in ATP synthesis. The endoplasmic reticulum is divided into two categories; rough and smooth. The ER plays a role in synthesizing and exporting proteins.

Based on the diagram, descibe how the structure of these organells controls movement across the membranes.

The molecules would first pass through the first membrane and stop, or continue to pass through the second and third membrane.

Some plant species flower in response to increasing daily temperatures in the spring. Many of these species rely on pollinators that migrate based on changes in day length and the position of the Sun. The current global warming trend is placing new selective pressures on the species involved in these relationships. Which of the following best explains the impact of these new selective pressures on the organisms involved?

The plant species will flower earlier in the spring in response to rising temperatures before the arrival of the pollinators, so seeds will not be produced.

The researchers determined that the East lava flow population is not in Hardy-Weinberg equilibrium. Which of the following best explains why the population is not in Hardy-Weinberg equilibrium?

The presence of MC1Rd alleles in the East lava flow population is an indication of gene flow from mice populations living on light granite rocks.

A molecule of messenger RNA has just been synthesized in the nucleus of a human cell. If the cell is a secretory cell, how is the protein eventually targeted, packaged, and secreted to the exterior if the cell?

The protein needs an srp signal which will send it to the ER, and then the Golgi bodies, which will pack it away in the vesicles, which will transfer it to the cell membrane where it can exit through exocytosis

How is a proton (H+) gradient established and maintained by the electron transport chain and how many ATP molecules are produced by chemiosmosis?

The proton gradiant generated by proton pumping during the ETC is a stored form of energy. when protons flow back down, their concentration gradient's only route is through the ATP synthase (mitochondrial membrane). 38 molecules of ATP are produced by chemiosmosis.

The researchers predict that for any particular concentration of substrate, the C. aurantiacus enolase-catalyzed reaction is more rapid at 55°C than at 37°C. Provide reasoning to justify the researchers' prediction.

The response indicated that the enolase has a more stable/functional/correct/normal protein structure at the higher temperature of 55°C than at 37°C because the enzyme is from an organism that is adapted to growth at 55°C.

Using the template below that represents the four levels of protein structure, place an X on each level that is expected to be altered as a result of the amino acid substitutions shown in Figure 1.

The response indicates primary and tertiary structure.

(a) Based on the data in Table 1, identify the animal that has the greatest number of sequence differences from the reference animal.

The response indicates that Elephas-2 has 13 sequence differences from the reference animal, and this is the greatest number of all animals in the table.

Describe the biological need for cells to be surrounded by a membrane that is selectively permeable for different materials.

The response indicates that a cell membrane separates the internal environment of the cell from the external environment OR that a cell membrane that is selectively permeable can help to regulate what molecules move between the differing environments inside and outside of a cell.

Explain how the model shows selective permeability of the membrane to specific ions.

The response indicates that a membrane channel protein is selective for a specific kind of ion because only one of the two kinds of ion represented can pass through the channel. The response also indicates that ions cannot simply cross directly between the phospholipids of the membrane (although a small lipid-soluble molecule can).

Depending on the organism, the optimal pH for enolase to catalyze its reaction is between 6.5 and 8.0. Describe how a pH below or above this range is likely to affect enolase and its catalytic ability.

The response indicates that a pH below or above this range will most likely cause enolase to denature/change its shape and be less efficient or unable to catalyze the reaction.

(d) There are six living species of mammals called hyraxes in the order Hyracoidea. These are small rabbit-like mammals that are also considered to be close relatives of both dugongs and elephants. All of these mammals are widely different morphologically. Explain how organisms that are widely different in morphology can have a close evolutionary relationship.

The response indicates that animals that are related once had a common ancestor with certain genetic characteristics. Adaption to different habitats leads to diversification of morphology but does not change evolutionary relationships.

(b) Based on the data in Table 1, complete the cladogram using the template provided to indicate the evolutionary relationships of the four species: African elephants (Loxodonta africana), Asian elephants (Elephas maximus), woolly mammoths (Mammuthus primigenius), and dugongs (Dugong dugon).

The response indicates that from left to right, the order at the tips of the cladogram is: Dugong, Elephas, Loxodonta, Mammathus. Loxodonta and Mammuthus can also be reversed. (Common names can be used; full species names can also be used).

(c) Based on morphological and molecular data, researchers hypothesize that the split between dugongs and proboscideans likely occurred between 48 and 34 million years ago. Explain how molecular data from fossils and living organisms would support the existence of an evolutionary relationship between dugongs and proboscideans.

The response indicates that molecular data, such as that for a widely conserved protein such as cytochrome b, show conserved similarities between organisms such as dugongs and proboscideans and can be used to support the existence of this relationship.

Provide reasoning with evidence based on the composition of biological macromolecules to support your claim.

The response indicates that nucleic acids (DNA, RNA, or nucleotides) and amino acids (polypeptides, or proteins) contain nitrogen.

Describe the role of oxygen in cellular respiration.

The response indicates that oxygen is the terminal acceptor of electrons that have passed through the electron transport chain.

Identify the most likely mode of transport across the membrane for substance L. Explain how information provided helps determine the most likely mode of transport.

The response indicates that substance L most likely crosses the membrane by active transport. The response indicates that the cells maintain an intracellular concentration of 120 mM, which is always greater than the concentration of substance L in the culture medium.

Predict the likely effect on the ability of substance L to enter the cells if substance L is attached to a large protein instead of free in the culture.

The response indicates that substance L will be unable to enter the cells if it is attached to a large protein OR the response indicates that substance L will be unable to enter the cells if it is attached to a large protein unless there is a specific cell membrane receptor for the protein.

Identify the appropriate negative control the researchers most likely used when measuring the reaction rate in the presence of each organism's enolase.

The response indicates that the appropriate negative control is to measure the reaction rate (at the varying substrate concentrations) without any enzyme present.

Make a claim about the most immediate effect on the fused B-cancer cells if the fused cells are transferred to a growth medium that lacks a source of nitrogen.

The response indicates that the cells will die OR that they will be unable to synthesize DNA and RNA/nucleic acids/nucleotides and amino.

Describe the relationship between the concentration of glucose in the culture medium and the ATP concentration in the cells.

The response indicates that the concentration of glucose appears to have little or no effect on the ATP concentration of the cells.

Determine the external concentration of substance L that will result in one-half of the maximal entry rate.

The response indicates that the external concentration of substance L that will result in one-half of the maximal entry rate is a value other than one between 31 mM and 34 mM.

Describe the characteristics of the phospholipid bilayer that permit small hydrophobic lipid molecules to pass directly across the membrane.

The response indicates that the interior of a phospholipid bilayer is hydrophobic, which allows small hydrophobic molecules to diffuse across the membrane.

Explain how, when the membranes are fused, the polar parts of the phospholipids from one cell will interact with the phospholipids from the other cell and how the nonpolar parts of the phospholipids from one cell will interact with the phospholipids from the other cell.

The response indicates that the polar part of the phospholipids from one cell will align or interact with the polar parts of the phospholipids from the other cell, and nonpolar parts of the phospholipids from one cell will align or interact with nonpolar parts of the phospholipids from the other cell.

Identify the process used to form the covalent peptide bonds that join amino acids into a polypeptide.

The response indicates that the process that joins amino acids into a polypeptide is a dehydration synthesis OR a condensation reaction.

The change in the amino acid sequence illustrated in Figure 1 caused a change in the shape of Receptor X. Based on the R groups of the original and substituted amino acids, explain why Receptor X changed shape.

The response indicates that two amino acids with charged or ionic or hydrophilic R-groups were replaced by two amino acids with uncharged or nonpolar or hydrophobic R-groups.

9. In mice, allele 1 codes for white fur and allele 2 codes for black fur. Describe the resulting phenotypes and phenotype ratios of a cross between two heterozygous F1 hybrid mice under all of the different possible patterns of inheritance.

The result of an F1 dihybrid cross of two heterozygous creatures is a phenotypic ratio of 9:3:3:1. Mice possess the alleles for both white and black fur. 9 offspring will possess a dominant allele for each trait, 3 offspring will possess a dominant allele for one trait, 3 offspring will possess a dominant allele for the other trait, and 1 will possess no dominant alleles, just recessive. The pattern of inheritance-polygenic Polygenic inheritance because the phenotype is under the control of multiple factors; in this case, black and white alleles.

(a) Describe how the structure of a prokaryotic chromosome differs from the structure of a eukaryotic chromosome.

The structure of prokaryotic chromosome differs from the structure of eukaryotic chromosome by following ways: -A prokaryotic chromosome is shorter than the eukaryotic chromosome. -In prokaryotes, the circular chromosome is held in the cytoplasm in a region called nucleoid whereas in eukaryotes chromosomes are stored within a structure called the nucleus. -Prokaryotic chromosome comprises only a single origin of replication whereas the eukaryotic chromosome carries multiple origins of replication. -Prokaryotic chromosome consists of covalently closed (loop) circular DNA whereas the eukaryotic chromosome consists of linear DNA not circular.

What is translation?

The synthesis of a polypeptide using the genetic information encoded in an mRNA molecule

Identify the THREE major stages of interphase, and describe the sequence in which they occur.

The three major steps or phases of interphase are G1, DNA Synthesis (S), and G2. In the G1 phase growth of the cell occurs. In the DNA Synthesis (S) phase the DNA is copied. In the G2 phase organelles are made and the final preparations are made for mitosis.

What are the major structures of prokaryotic cells?

There are four main structures shared by all prokaryotic cells, bacterial or archaean: The plasma membrane, cytoplasm, and ribosomes.

How does the structure of the eukaryotic cell resemble as well as differ from the structure of the prokaryotic cell?

There are several differences between the two, but the biggest distinction between them is that eukaryotic cells have a distinct nucleus containing the cell's genetic material, while prokaryotic cells don't have a nucleus and have free-floating genetic material instead.

What are functions of proteins in cells and tissues?

They do most of the work in cells and are required for the structure, function, and regulation of the body's tissues and organs. Proteins are made up of hundreds or thousands of smaller units called amino acids, which are attached to one another in long chains.

How do electrochemical gradients affect the active transport of ions and molecules across membranes?

To move substances against a concentration or electrochemical gradient, the cell must utilize energy in the form of ATP during active transport.

What is tonicity, and how is it relevant to passive transport?

Tonicity is the amount of solute in a solution. This is relevant to passive transport because passive transport is the ability to move a solution through a membrane without energy, this involves osmosis as well.

Differences between transcription and translation

Transcription: products are mRNA, tRNA, rRNA, and non-coding RNA; take place in the nucleus Translation: product is protein; takes place in the cytoplasm

How is pyruvate, the product of glycolysis, prepared for entry into the citric acid cycle?

Two molecules of pyruvate were produced from glycolysis and converted into two molecules of acetyl coenzyme A, two carbon dioxide, and two NADH + H+ molecules through pyruvate oxidation, an intermediate step between glycolysis and the citric acid cycle.

Connect primary structure of an enzyme to its overall shape

The primary structure of an enzyme is the sequence of amino acids linked by peptide bonds in a linear chain. The resulting amino acid chain is called a polypeptide or protein. Interactions between the amino acids determine the enzyme's secondary structure, being either alpha helices or beta pleated sheets. The tertiary structure is the enzyme's overall three dimensional structure, which is primarily due to interactions between the R groups of the amino acids that make up the protein. This demonstrate how the primary structure has a huge impact on the enzyme's overall shape.

Discuss THREE properties of water

Water is polar, slightly charged on both ends, because oxygen is more electronegative than hydrogen. Water's cohesion allows for surface tension, which allows animals like the basilisk lizard can walk on water. Water has a high specific heat, which allows for evaporative cooling to occur.

What is the relationship between amino acids and proteins?

When connected together by a series of peptide bonds, amino acids form a polypeptide, another word for protein. The polypeptide will then fold into a specific conformation depending on the interactions (dashed lines) between its amino acid side chains.

The arrangement of the water molecules in ice causes the ice to float. Explain how ice floating on the surface of a body of water affects the water in a way that is beneficial to the organisms in it.

When water freezes in the winter, floating ice forms a layer of insulation on top of the water. The ice layer insulates the water below it, allowing it to stay liquid, therefore allowing life within the water to survive.

5. The quality of light influences many processes in organisms, including photosynthesis in plants. The absorption spectrum for chlorophyll a and the action spectrum for photosynthesis in spinach leaves are shown below. a)Explain the relationship between the absorption spectrum of chlorophyll a and the action spectrum of photosynthesis. b) Using specific data from the absorption spectrum of chlorophyll a, explain why the leaves of most photosynthetic plants are green. c) Explain why photosynthesis in plants occurs in areas of the light spectrum where chlorophyll a does not absorb light.

a) Action Spectrum of Photosynthesis follows Absorption Spectrum of Chlorophyll. The absorption spectrum indicates how much of each wavelength chlorophyll will absorb, whereas the action spectrum can tell us which off those wavelengths are most effective in photosynthesis. b) Because most photosynthetic plants use chlorophyll a which absorbed red and blue like and reflects green like so the leaves are green. c) Chlorophyll a is not the only pigment, caratoid blocks excess light

4. Both cellular respiration and photosynthesis produce energy for an organism. Compare and contrast the processes of energy production in cellular respiration and photosynthesis in electron transport and chemiosmosis.

a) Cellular Respiration 1. Glycolysis 2. Krebs Cycle 3. ETC and ATP Synthase b) Photosynthesis 1. Light reactions -PS I and PS II 2. Calvin cycle

1. Membranes are important structural features of cells. a) Describe how membrane structure is related to the transport of material across a membrane b) Describe the role of membranes in the synthesis of ATP in either cellular respiration or photosynthesis.

a) During the ETC, proteins are embedded within the membrane and electrons go through, which pumps H+ ions into the intermembrane space. b) During photosynthesis, water molecules are broken in PS II and then as they travel through the membrane they pump protons into the thylakoid space, which leads them to ATP Synthase.

8. This graph shows the amount of oxygen consumed for Organism A at various environmental temperatures. Oxygen consumption is on the Y axis and temperature in degrees Celsius is on the X axis. a) Is this organism and ectotherm or an endotherm?

a) Ectotherm b) When the temperature increases, so does the metabolism, which in turn requires more oxygen to be consumed, as it is needed as the final electron acceptor.

6. Enzymes are biological catalysts. a) Relate the chemical structure of an enzyme to its specificity and catalytic activity. b) Design a quantitative experiment to investigate the influence of pH, temperature, or enzyme concentration on enzyme activity. c) Describe what information concerning the structure of an enzyme could be inferred from your experiment.

a) Enzymes need a lock and key model -High specefficity -Lock and key mofel -Substrate binds to active site b) Denature is the temperature is too high c) Explain denaturing and how the environment has to be super specific

10. Photosynthesis and cellular respiration recycle oxygen in ecosystems. Respond to the following: a) Explain how the metabolic processes of cellular respiration and photosynythesis recycle oxygen. b) Discuss the structural adaptations that function in oxygen exchange between each of the following organisms and its environment: a plant; an insect; a fish.

a) Oxygen is used as the final electron acceptor in electron transport chains in cellular respiration. This is then released as H2O. The H2O is used in photosynthesis to release electrons, where oxygen is split off of the hydrogen, and oxygen is expelled. b) Plant- Photosystem II splits H2O Insect- gets it from the atmosphere Fish- through gills and H2O

FRQ #7: a. The evolution of a species is dependent on changes in the genome of the species. Identify TWO mechanisms of genetic change, and explain how each affects genetic variation. (4 points maximum) c. Describe TWO types of evidence—other than the comparison of proteins—that can be used to determine the phylogeny of organisms. Discuss one strength of each type of evidence you described.

a. 1) DNA -Mutation: Change in nucleotide sequence -Duplication: Gane "families", which diverge by mutation 2) Population -Genetic drift: Population allelic frequencies altered -Gene flow: Reproductive mating differential success c. 1) Fossil- observe past organisms -shows direct evidence and follows evolution from common ancestor 2) Homology- organismal structure/form -show common ancestry through DNA

FRQ #6: a. What is the frequency of each genotype (AA, Aa, aa) in this population? What is the frequency of the dominant phenotype? b. How can the Hardy-Weinberg principle of genetic equilibrium be used to determine whether this population is evolving? c. Identify a particular environmental change and describe how it might alter allelic frequencies in this population.

a. -Frequency AA = .36 -Frequency Aa = .48 -Frequency aa = .16 -Frequency dominant phenotype = .84 b. Evolving population -ALlelic frequency changes -Five conditions that do not change if population is evolving c. Tornado -New allel frequencies

FRQ #1 a. Using a specific example, describe how organisms can reproduce asexually. Discuss TWO evolutionary advantages of asexual reproduction. b. Identify THREE ways that sexual reproduction increases genetic variability. For each, explain how it increases genetic diversity among the offspring. c. Discuss TWO prezygotic isolating mechanisms that prevent hybridization between two species. Include in your discussion an example of each mechanism.

a. 1) Specific Example: Bactaria -Binary fission splits into two cells Evolutionary Advantages: 1. It is successful at low population density 2. Eliminates energy cost of finding a mate b. 1. Crossing over -Generates new combinations of alleles 2. Independent Assortment -Random alignment on metphase plate during meiosis 3. Random fertilization -nonspecific gamete seletion c. 1. Habitat/ecological isolation -Preferences for living /mating in different habitats 2. Geographic isolation -Living or mating in different geographic areas with a physical barrier

FRQ #2: a. Describe the change in genetic variation in the population between 0 and 6 months and provide reasoning for your description based of the means and SEM b. Propose ONE type of mating behavior that could have resulted in the observed change in the number of spots per adult male guppy between 6 and 20 months in the absence of the predator. c. Propose an evolutionary mechanism that explains the change in average number of spots between 6 and 20 months in the presence of the predator.

a. 1. Genetic Variation is decreasing 2. SEM gets smaller b. 1. Sexual selection for individuals with more spots 2. Random mating behavior resulted in increased nuber of spots by chance c. Natural selection ex:/ spots not ound survived

FRQ #5: a. Discuss TWO mechanisms of speciation that lead to the development of separate species from a common ancestor (b) Explain THREE methods that have been used to investigate the phylogeny of organisms. Describe a strength or weakness of each method.

a. 1. Geographic Isolation: takes place when a population od one species becomes physically separated by a barrier 2. Reproductive Isolation: Prezygotic barriers, such as habitat temportal, behavioral b. Method 1: Fossils Pro: Determine time Con: Not all species leave fossils Method 2: Anatomy Pros: Indicate evolutionary relationships Cons: SImiarities between species are lost during later development Method 3: Behavioral traits Pros: Some behaviors are genetic COns: Behavior may be culturally transmitted or learned

2000 Information transfer is fundamental to all living organisms. For two of the following examples, explain in detail how the transfer of information is accomplished. a. The genetic material in one eukaryotic cell is copied and distributed to two identical daughter cells b. A gene in a eukaryotic cell is transcribed and translated to produce a protein.

a. DNA copied, helicase unzips/unwinds, RNA primer, semiconservative relationship among bases; P: condensed chromosomes M: alignment to the middle A: chromatids pulled to opposite poles T: cell cleavage furrow b. - Transcription: DNA to RNA, promoter recognition, complementary bases, 5' to 3' - Translation: base sequence to amino acid sequence, complementary (codon to anticodon), peptide formation (aa joined by peptide bonds = polypeptide), stop codon + release of polypeptide

FRQ #3: a. Select the most appropriate sample of rocks in which to search for a transitional species between lobe-finned fishes and amphibians. Justify your selection. b. Describe TWO pieces of evidence provided by fossils of a transitional species that would support a hypothesis that amphibians evolved from lobe-finned fishes.

a. Selection: rocks from 370 MYA sample Jutification: Transitional fossils are found between 380 MYA and 363 MYA b. 1. Bones or specific skeletal structure -Left limbs 2. Molecular DNA

FRQ #4: a. Assuming that species I is the ancestral species of the group, explain the most likely genetic change that produce the polypeptide in species II and the most likely genetic change that produced the polypeptide in species III. b. Predict the effects of the mutation on the structure and function of the resulting protein in species IV. Justify your prediction.

a. Species II: Mutation/Substitution/Point Mutation -An amino acid change only at position 4 Species III: Mutation that introduces a stop codon after the codon for Val -Termination of the polypeptide after the Val at position 8 b. Predicted Change: PRotein may have a different structure and change in function Justification: Change in amino acid sequence of the protein starting at position 5 could alter the overall structure

pH

adds or removes H+ and disrupts bonds, 3D shape, attractions bt charged amino acids

salinity

adds or removes cations and anions -disrupts bonds, 3D shape, attractions bt charged amino acids

Describe structural changes that can occur in a protein after translation to make it function properly.

after translation the amino acid sequence can fold to form the secondary or tertiary structure.

Mechanisms that disperse offspring (FLOWERS)

animals eating fruits

11. The results below are measurements of cumulative oxygen consumption by germinating and dry seeds. Gas volume measurements were corrected for changes in temperature and pressure. a) Using the graph paper provided, plot the results for the germinating seeds at 22o C and 10o C. b) Calculate the rate of oxygen consumption for the germinating seeds at 22o C, using the time interval between 10 and 20 minutes.

b) new-old/# of minutes

similarities between transcription and translation

base pairing differs, in transcription dna to rna is paired for example in dna AAA turns into UUU. in translation base pairing is mrna to trna (codon-anti codon), for example AUG turns into UAC. another similarity is that they both have specific start sites, in transcription there is the promoter or tata box and translation has the start codon AUG. Both have specialized proteins for their jobs, EX- transcription=rna polymerase, translation= initiation factors

two types of reactions

catabolic: (hydrolysis) breaking bonds between molecules to release energy -exergonic anabolic: (dehydration/protein synthesis) forming bonds bt molecules and consuming energy -endergonic

membranes in the following biological processes (chemiosmotic production of ATP, intercellular signaling)

chemiosmotic: H+ pumped across membrane, establishing H+ gradient, H+ move thru ATP synthase and thus produce ATP intercellular: receptors in membrane bind to ligands and let them pass thru the membrane; ligand-gated ion channel receptors have a region that is a gate for ions and lets them pass thru when the receptor is a certain shape

prokaryotic chromosomes

circular, nonlinear, less complex, generally smaller, less genetic information, replicate via binary fission at origin of replication

What are the two types of nucleic acid?

deoxyribonucleic acid (DNA) and ribonucleic acid (RNA).

osmosis

diffusion of water, high to low, thru lipid bilayer bc it allows the passage of small polar molecules and is very fluid. allows for water to get into the cell and support life and reach needed level of tonicity

describe transcription

dna is temporarily unwound and an rna polymerase begins to make a new complementary mrna strand starting after the promoter and the tata box. after the mrna strand is completed a 5' cap is added to the 5' end and a poly-a tail is added to the 3' end. the rna then leaves the nucleus and enters the cytoplasm for translation.

germ cell's complement of chromosomes is halved in the formation of gametes (aka meiosis)

meiosis: duplication of chromosomes by two consecutive cell divisions (i and ii) resulting in four daughter cells. prophase i- homologous chromosomes formed; metaphase i- line up at plate; anaphase i- separate into sister chromatids; telophase i- two seperate haploid cells; dna does not replicate again here, same things but with sister chromatids who separate into four cells

A classmate proposes that mitochondria and chloroplasts should be classified in the endomembrane system. Argue against the proposal.

mitochondria and chloroplasts are not derived from the ER and not connected physically or via transport vesicles to organelles of the endomembrane system mitochondria and chloroplasts are structurally quite different from vesicles derived from the ER which are bounded by a single membrane

inhibitors

molecules that reduce enzyme activity -competitive inhibition, noncompetitive inhibition, irreversible inhibition, feedback inhibition

differences in plant diversity in the graph and the affect of animal species composition

more diversity in ground flora and shrubs, less in understory and canopy; ground flora/shrubs diversity grew at a higher rate pioneer stage: small animals and insects spruce stage (final): large animals and a diverse, multilayered community of fauna

temperature

optimum temp = greatest # of molecule collisions -heat increases beyond optimum T: increased energy level of molecule disrupts bonds in enzyme and substrate -results in denaturation: loses 3D shape

seedlings placed in a chemical that prevents NADH from oxidizing to NAD+: effect on metabolism and oxygen consumption

oxygen consumption and metabolism decrease, bc cellular respiration stops and so ATP levels decrease. oxygen cannot accept electrons for the electron transport chain, as well

Hardy-Weinberg equation

p+q=1

Hardy-Weinberg equation

p2+2pq+q2

three bonds in proteins

peptide: covalent bond btwn carboxyl group and amino group in a dehydration reaction producing water as a result, long chain of aa=primary structure hydrogen bonds: secondary structure (backbone interactions), interactions btwn the repeating parts of the polypeptide backbone; hb is when an electronegative element forms a weak bond w H disulfide bridges: covalent bond btwn two cysteine monomers which have sulfhydryl groups, tertiary bond (interactions btwn R-group), S-S bond very tight

occurring to the population during phase A (but also the others)

phase a: establishment period, slow growth rate eventually it will begin to grow exponentially, a period of rapid growth finally population growth slows as carry capacity is reached- growth rate leveling off

structure of a protein affects regulation of enzymatic activity and cell signaling

regulation of enzymatic activity: enzyme requires a very specific shape, shape change can occur w feedback inhibition (signals back to the beginning to inhibit reaction and stop it from being wasteful) or binding of an allosteric inhibitor (binding of a regulatory molecule to a separate site that switches off activity) cell signaling: receptor protein needs a specific ligand; ligand-gated channel- bind of ligand opens the channel; receptor-ligand binding- ligand binds to receptor and alter the receptor's structure which transduces a signal thru the membrane

ATP -> ADP

releases energy as phosphate is removed to fuel other reactions

imagine an elongated cell that measures 125 by 1 by 1 unite4s. predict how the surface volume ratio compare with 4.6 calculate ratio

same volume as cells in 2 and 3, more surface area than 2 and less than 3 add six sides and then 502 divided by the volume of 125 which is 4

fossil evidence for evolution

shoulders & wrist

If the plant cell wall or the animal extracellular matrix were impermeable, what effect would this have on cell function?

the cell could not function properly, and would die, cell wall or ECM has to be permeable for exchange of matter, molecules in energy production and use must have entry, and those that provide info about the environment, and other molecules like byproducts of cellular respiration must be allowed to leave

describe translation

the completed mra strand moves into the cytoplasm and binds to a ribosome and a trna for translation, this stage is initiation. the next stage is elongation where amino acids created from the codons are linked together by the trna to form a polypeptide chain. the final stage is termination where the polypeptide is released.

If any ionization increases... (use the first)

the water potential decreases


Kaugnay na mga set ng pag-aaral

Software & Hardware Interaction lesson 6

View Set

Alteration in Arterial/Vascular Function

View Set

Pharmacology Chapter 7, Over-the-Counter Drugs and Dietary Supplements

View Set

MUS 185: Latin American/Caribbean Unit

View Set

Revel Ch.15 Sph: Tools for Persuading

View Set

Servers Cha 2- Configuring Windows Server 2019

View Set

Chapter 2: The Rise of Astronomy

View Set

microeconomics comprehensive review

View Set